LSAT For Dummies, 2nd Edition (2014)

Part VI. The Real Deal: Full-Length Practice LSATs

Chapter 20. Practice Exam 3: Answers and Explanations

This chapter has answers and explanations for the multiple-choice sections in Chapter 19. Use the abbreviated key at the end of this chapter to score your test quickly, but be sure to read through the answer explanations for the questions you miss. For maximum benefit, read the explanations for all the questions. Your essay won't be scored, but compare your version to the example provided and examine your response through the eyes of a law school admissions officer. Does your essay provide a clear, well-supported position? Is it well-organized and free of glaring grammar, usage, and spelling errors? If you can answer yes to these questions, you've met the requirements of the exercise.

Section I: Logical Reasoning

1. D. Of the five key field marks that identify ivory-billed woodpeckers, only the extra white on the wings has been seen, and this is also the only feature that occurs on abnormal pileated woodpeckers.

This question concerns the reported discovery of ivory-billed woodpeckers in Arkansas. The argument the skeptics challenge is that, because the sighted birds have large white patches on their wings and ivory-billed woodpeckers have large white patches, the sighted birds are ivory-billed woodpeckers. The skeptics argue that an abnormal pileated woodpecker may also have the distinctive white patches. You're asked to strengthen the skeptics’ conclusion, so you need an answer choice that strengthens the idea that the white patches on the sighted birds are abnormalities of pileated woodpeckers.

Choice (B) doesn't have much effect on the argument; the skeptics don't have an issue with the credentials of those who confirmed the identification. Choices (C) and (E) each weaken the skeptics’ conclusion: Choice (C) indicates that researchers heard the audio sign of the ivory-billed — the distinctive double-tap — and Choice (E) indicates that the wing beats on the video are indicative of the ivory-billed rather than the pileated.

Make sure you remember which conclusion you're strengthening. The question requires you to support the skeptics’ conclusion rather than the researchers’ claim, so you can eliminate Choices (C) and (E), which support the idea that the bird may be an ivory-billed woodpecker.

You're left with Choices (A) and (D). You can eliminate Choice (A) because the fact that an ivory-billed was last seen in Louisiana doesn't necessarily mean one couldn't be in Arkansas. Choice (D) is the best answer of those available. The fact that the only sighted field mark of ivory-billed woodpeckers also happens to be the only one that's a feature of an abnormal pileated would support the skeptics’ conclusion that the bird is more likely an abnormal pileated woodpecker than an ivory-billed variety.

2. D. The women who had used shampoo for color-treated hair had softer and shinier hair than the women who used ordinary shampoo.

The question wants you to weaken the argument. The argument concludes that shampoo for color-treated hair is valueless. The evidence for this claim is that colored hair washed with shampoo for color-treated hair faded as much as colored hair washed with normal shampoo. What weakens the claim that special color shampoo is valueless? Evidence of its value in some measure other than holding hair's color — so look for an answer that shows that this kind of shampoo does do something desirable that ordinary shampoo doesn't do.

Choice (A) is irrelevant to the argument; it's just a statement of ingredients and doesn't address the quality or performance of either kind of shampoo. Choices (B), (C), and (E) also have nothing to do with the relative performance of the two types of shampoos. Choice (D) is the only answer choice that addresses the possible value of shampoo for color-treated hair. If it makes hair softer and shinier than ordinary shampoo, then it does have some value, even if it doesn't preserve hair dye any better than ordinary shampoo. All you have to do is weaken the argument; you don't have to prove that it's wrong. Choice (D) is the best choice for that.

3. E. Snakes are also absent from other major islands, such as Hawaii, Ireland, and Greenland.

This question discusses the fact that no snakes live on the island nation of New Zealand. It asks you to strengthen the conclusion that the inability of snakes to swim is responsible for their absence. To strengthen the cause-and-effect argument, find an answer that promotes the lack of swimming as the primary cause of the lack of snakes. Choice (B) doesn't address the snake-swimming issue, so it doesn't strengthen the argument. There's no question of whether snakes exist in the areas around New Zealand. Choice (C) helps explain why snakes haven't been accidentally introduced in New Zealand, but it doesn't say anything about why they weren't on the island in the first place. Choice (D) discusses sea snakes, which are different from the land snakes the paragraph talks about, so their characteristics aren't applicable to the author's conclusion.

Choice (A) weakens the conclusion; if snakes appear on other islands, they must have gotten there somehow, regardless of whether they could swim. Choice (E) is the best answer of the five choices, because if snakes are absent from other large islands, the reason would seem to be that they can't swim. Don't allow your knowledge that snakes exist on islands like Hawaii stop you from choosing Choice (E). Remember that you're supposed to assume the answer choices are true regardless of whether you know otherwise.

4. C. Dirt on the skin doesn't cause acne.

The question asks you to find the assumption that the dermatologist must make in order to make the conclusion follow logically. The conclusion is that external treatments have no effect on acne. The evidence for this conclusion is that acne develops within the skin. The unstated assumption must be that nothing on or outside the skin's surface can cause that development within the skin; see if that's one of the answers.

Choice (A) is wrong because the dermatologist isn't critiquing the side effects of external acne treatments, just their effectiveness. Choice (B) is wrong because the sensation caused by toners has nothing to do with the causes of acne or the effectiveness of toners as a treatment. Choice (C) looks like the answer you want. If dirt on the skin doesn't cause acne, and it's only caused by internal sources, then Choice (C) justifies the doctor's claim that toners and masks that remove dirt or keep it out of pores don't work. Choice (D) seems to support the doctor's conclusion that acne is an internal problem; the success of oral antibiotics in treating acne doesn't necessarily rule out external factors (like dirt) in causing acne. Choice (E) may be true, but the dermatologist doesn't need to assume it to reach the conclusion. In fact, you could conceivably use Choice (E) as evidence against the conclusion. So the answer is Choice (C).

5. C. The United States is said to have a free market economy. In fact, though, the U.S. economy is heavily regulated by the government, which gives many advantages to large businesses and punishes small companies and the self-employed. Therefore, the United States doesn't really have a free market economy.

The question asks you to identify the pattern of reasoning and match it with an answer. The argument's pattern of reasoning goes like this: Although people think the United States is a democracy, citizens don't actually vote for president, so it's not a democracy. You can simplify the reasoning pattern: People think A, but here's an example of something that runs counter to A, so A isn't the case. Questions like this can be a bit time-consuming because you have to work out the thought pattern in all the answers. So forge ahead!

You could summarize Choice (A) like this: Most colleges have A, but few students participate, so A is useless; that's not the same. Choice (B) reasons that all cities have A, but so do counties and states, and that means A is needlessly duplicated. That's a different pattern. Choice (C) figures that the United States is said to be A, but here's an example of something that's not A, so A isn't really the case. That looks pretty close to the original argument's reasoning, so Choice (C) could be the answer. Choice (D) says that most teachers think A, but colleges disagree, so teachers shouldn't do A. That's not quite the same. Choice (E) goes like this: Most insurers claim A, but doctors and hospitals don't, so insurers are dishonest. That's way off. Choice (C) is by far the best choice, so it's your answer.

6. A. A teacher informs a girl that she is responsible for feeding the class hamster every morning, and the girl agrees to do this. After this, the girl feeds the hamster every morning without the teacher telling her to.

The question wants you to find an example that illustrates the proposition stated in the argument. That proposition is that people meet expectations. Look for an answer that shows someone receiving a task and performing it as expected. Choice (A) looks like a perfect choice: The teacher presents the girl with a task and expects her to do it, and the girl does. Choice (B) is wrong. In this case, the partner assigns the associate a duty but obviously doesn't expect the associate to be able to perform it and therefore supervises him closely. Choice (C) is an example of someone failing to meet expectations, so it's wrong. Choice (D) seems close to the proposition but doesn't fit it exactly because the original proposition regards doing one's assigned duty, not the success or failure of that duty. The players do learn the coach's strategy and execute it, but the scenario includes an additional result (they win) that isn't part of the original proposition; for this answer to fit the proposition, the players would instead merely execute the strategy as expected. Choice (E) doesn't involve anyone's expectations. The psychologist tells the patient something that may help her, and the patient follows the suggestion and succeeds. The psychologist's expectation is that the behavior modification technique, not the patient who follows it, will meet expectations, which is different from the original proposition. Choice (A) is the best answer.

7. A. Disciplinary and other problems are most profound at technical schools with large immigrant populations, where students feel that society will not give them opportunities for self-improvement.

The question informs you that four of the answer choices weaken the argument; only one doesn't. The argument's conclusion is that aptitude tests have no value in sorting students. As evidence, it cites the problems that currently plague German schools, which have used these tests for more than a century. What weakens this argument? Well, if the argument blames aptitude-test-based sorting for the problems in German schools, anything that shows that the problems aren't due to the sorting or the tests weakens it. When you read the answers, look for four choices that do this. The correct answer doesn't. It doesn't necessarily strengthen the argument, but it doesn't weaken it.

Choice (A) doesn't obviously weaken the argument. If disciplinary problems are worst at technical schools and the cause is that students fear they won't ever receive the opportunity to improve, perhaps the sorting system is the root of the problem. You could argue that aptitude-test-sorting schemes are explicitly designed to restrict opportunities. The sorting can have benefits, but perhaps in this case, it also causes problems, such as resentment on the part of those students who get sorted into the lower echelons. Choice (B) weakens the argument by showing a reason why sorting students by ability is important, citing the problems that occur when students of mixed abilities end up in the same classroom. Choice (C) weakens the argument by claiming that schools that sort students by ability get better results. Choice (D) weakens the argument by claiming that the sorting system worked fine for more than 90 years. If the problems have only arisen in the last decade, perhaps some other cause is to blame. Choice (E) weakens the argument by stating that technical schools are actually very good at training the students who get sorted into them, suggesting a strength of the sorting method. Choice (A) is the only choice that doesn't weaken the argument, so it's the answer.

8. B. When a hive loses its queen, some workers develop the ability to lay eggs; these unfertilized eggs can hatch only into male drones.

The question asks for a statement that weakens the argument. The argument's conclusion is that worker bees never pass on their own genes. The evidence is that they put all their energy into nurturing the queen and her offspring. To weaken this argument, you need a statement that shows that despite their devotion to the queen, workers can pass on their own genetic traits. Choice (A) seems like it could be the answer because it suggests that workers are, in fact, furthering their genetic line by caring for the queen, who is, after all, related to them. But it doesn't quite work because it doesn't contradict the argument, which says that workers fail to pass on their own genes to offspring. See if a better choice is available. Choice (B) contradicts the argument's conclusion: If workers can, in fact, lay eggs, they do occasionally pass their genetic traits on to offspring, even if these offspring are exclusively male. Choice (C) doesn't weaken the argument, but it doesn't strengthen it, either. It's merely a statement of fact about queens. Choice (D) doesn't weaken the argument; if anything, it shows how workers subordinate their interests to the queens. Choice (E) is just more information about queens (and drones); it neither strengthens nor weakens the argument. Choice (B) is the best answer.

9. E. assumes that what is true of myths in general must also be true of each individual myth

The question asks you to find the argument's weak point. The argument concludes that all myths can fit into both categories (explanations of natural phenomena and explorations of psychological phenomena) based on the evidence that those are two general functions of myths. Choice (A) is wrong. You don't know if experts in the field have already discounted this conclusion because the argument doesn't say anything about them. Choice (B) is wrong because the argument says nothing about the relevance of myths in today's world. Choice (C) is wrong because the argument doesn't impose any sort of modern sensibility on myths; it doesn't mention pre-modern people or the difference between them and people today. Choice (D) is wrong because the argument doesn't criticize anyone. Choice (E) matches with the answer you hoped to find; the argument does indeed assume that all myths contain both natural and psychological phenomena and doesn't offer any proof that those general characteristics of myths are universal features of each and every myth. Choice (E) is the answer.

10. A. The sex of a non-juvenile parrotfish lacking brightly colored scales cannot always be easily determined.

The question asks you to find a statement that must be true if everything in the argument is true. The passage states that identifying the sex of parrotfish can be difficult, and it cites the various color patterns that male and female parrotfish display at different stages in their lives. You know enough information is available for you to infer more facts, but at this point, you don't know what, so why try speculating? You can't predict what the answer will be, so just read the statements carefully and get your head around the parrotfish facts. Then just read the answer choices one by one and see if one fits what you now know about these colorful fish.

Right off the bat, Choice (A) looks like a good choice. Because some post-juvenile but pre-sexually mature male parrotfish continue to look like females, it won't always be easy to identity the sex of a non-juvenile parrotfish; in the intermediate growth phase, a male and female can look alike. Check the other answers to make sure this one is the best. Choice (B) can't be right because the argument tells you that juvenile male parrotfish look like the smaller, less colorful females; only the sexually mature males are the ones who are large and brightly colored. Choice (C) may be true, but the argument isn't about female parrotfish mating behavior, so it isn't relevant. Choice (D) may also be true, but the argument says nothing about the importance of camouflage in avoiding predators. Choice (E) is wrong; the argument says that in some species, males at some time look like females, but you can't infer any information about all male parrotfish from this statement. Choice (A) is your answer.

11. D. If people are not exposed to the arts at school, they will never learn about them.

The question wants you to spot the assumption that the author makes in developing the conclusion. Based on the premise that schools devote less attention to teaching the arts, the argument concludes that the next generation of citizens won't appreciate art. The only way that could be the case is if schools are the only place where children get any arts education. That's the author's assumption.

Choice (A) is wrong. If the author assumed that, then she couldn't conclude that everyone would grow up ignorant of the arts. The author doesn't assume Choice (B), because she is interested in arts education for its own sake, not for the sake of academic subjects. Choice (C) is wrong; the author certainly isn't assuming that people become great artists independent of art education in school. Choice (D) is right. The author definitely assumes that if people don't learn art in school, they'll never learn it. Choice (E) is wrong; it serves as an argument in favor of ending arts programs in schools. Choice (D) is the right answer.

12. B. The baseball team's presence generated millions of dollars of revenue for the town every year by attracting out-of-town visitors who spent money on hotels, restaurants, and other goods and services.

The question wants you to find an answer that supports the argument's conclusion. The conclusion is that the loss of the baseball team will hurt the town, especially economically. To support this, look for an answer that shows how much money the team brought to the town.

Choice (A) doesn't support the conclusion. In fact, it supports the opposite conclusion that keeping the team would actually cost the town money instead of bringing in income. Choice (B) does support the conclusion; it shows how the team's presence brought in money by bringing visitors to the town. Choice (B) looks like a good answer. Choice (C) provides another reason why losing the team wouldn't hurt the town; no one attended the games anyway. Choice (D) criticizes the baseball team's loyalty and its demands, which does nothing to support the argument that the town would benefit by keeping it. Choice (E) is evidence of the town's apathy toward the team, not support for an argument that says losing the team would hurt the town. Choice (B) is the only answer that actually supports the conclusion, so it's the best one.

13. C. Children raised by a mother and father who are married to one another are better off than children raised by a mother alone.

The question informs you that the argument makes an assumption and asks you to identify it. The argument concludes that the president's initiative is worth supporting because it will help the children of poor, unmarried women based on the evidence that the program will help these women get married and stay that way. Clearly, the writer assumes that married mothers benefit these children; if kids don't benefit, then the argument falls apart. Look for an answer that says something about the benefits kids get from having married parents. Choice (A) is wrong because the author says nothing about drug dependency, crime, or teen pregnancy. This argument sticks to statements about relationships and healthy children. A reader might assume that drugs and crime are part of the equation, but the author of this argument doesn't. Choice (B) doesn't work. The author doesn't need to involve people with higher incomes in the conclusion about marriage and low-income people. Choice (C) is exactly what you want. If children raised by married moms and dads are better off than those raised by single mothers, it justifies the author's conclusion. Choice (D) may be another reason to encourage marriage, but in this argument the author is talking only about children, not welfare. Choice (E) is irrelevant; the author makes no mention of other governmental programs, successful or otherwise, and doesn't need this statement to make the argument hold together. Choice (C) is the correct answer.

14. D. Isabel believes that popular fiction can affect people's beliefs and actions.

The question asks you to find a statement that the argument supports. (Note that it doesn't ask you to support either speaker; that would be hard to do with a conversation like this, which doesn't really have a conclusion.) Think for a moment about what's going on in the conversation. Isabel says that the novels in question are dangerous and explains why she thinks that. Ferdinand says he thinks she's wrong because she's exaggerating the influence the books have. So clearly they disagree about the impact these books have on their readers. See if one of the answers says this. Choice (A) is wrong. Ferdinand and Isabel don't argue about the religious nature of the books; Isabel mentions religion, but Ferdinand doesn't address that part of her contention. Choice (B) is definitely wrong. Isabel says nothing about the appropriateness of the end of the world as a topic for a novel; her focus is on the demonization of nonbelievers. Choice (C) is clearly wrong. Isabel and Ferdinand aren't arguing about whether the books are popular; they're arguing about whether popular fiction influences behavior. Choice (D) looks like a good choice. Isabel certainly does believe that popular fiction can affect people's behavior in a dangerous manner. This answer choice doesn't mention Ferdinand, but that doesn't necessarily mean it's wrong. Choice (E) is another wrong answer. Ferdinand and Isabel don't argue about the amount of violence that is appropriate in fiction. That leaves Choice (D) as your best choice.

15. D. Regulating modern food manufacturing and advertising would help prevent obesity.

This question wants you to identify the source of Tia and Muriel's disagreement. Why are they arguing? Tia thinks the government should regulate food manufacturing and advertising, and Muriel thinks it shouldn't, not necessarily because that regulation wouldn't work but because she thinks individuals are in control of their own weight. They also disagree about the causes of obesity; Tia seems to place the blame squarely on modern food manufacturers and advertisers, while Muriel blames the obese individuals for their bad habits. Choice (A) is wrong because Tia and Muriel aren't debating the effectiveness of diet and exercise in weight control; they both may well agree with the statement. Choice (B) is tricky. Tia obviously thinks the government should intervene in public health problems, while Muriel seems to disapprove of such intervention. But Muriel does say that the government could take some limited action in encouraging diet and exercise and says nothing about other situations, so Choice (B) is too extreme. Choice (C) is wrong; neither speaker has mentioned public schools, so there's no telling what they think about school nutrition programs. Choice (D) looks pretty good. Tia obviously thinks that a lot of obesity comes from modern food, but Muriel thinks it comes from overeating. Choice (E) doesn't work because neither Muriel nor Tia has mentioned the difference between public- and private-sector efforts (just because Muriel doesn't want the government involved doesn't necessarily mean she wants the private sector involved). Choice (D) is the best answer here.

16. D. relies on evidence too partial to establish the conclusion drawn

The question asks you to find the point where the politician's argument is most vulnerable. The politician suggests that global warming isn't happening. As evidence, he mentions only that two months of recent winters have set record low temperatures, which ignores that average annual temperatures take into account data from all twelve months. See if one of the answers refutes the validity of the politician's evidence. Choice (A) is wrong; the premise and conclusion are different statements, so the reasoning isn't circular. Choice (B) is wrong because the politician isn't appealing to emotions at all. Choice (C) is wrong because the politician doesn't debate the potential effects of global warming but whether it's even occurring at all. Choice (D) is right because the politician assumes that a few months of record low temperatures can refute the claim that overall temperatures worldwide are increasing. Choice (E) is wrong because the politician doesn't mention the political leanings of environmentalists. Choice (D) is your best answer.

17. A. A college graduate with a lucrative job offer decides to decline it and instead borrow several thousand dollars to attend medical school in the hopes of receiving a much larger future income as a physician.

The question asks you to identify the answer that conforms to the principle stated in the argument. What is that principle? Basically, the argument suggests that investing for the future is better than taking immediate profits because the long-term profits from the investment will ultimately be larger. Look for an answer that shows someone postponing gratification or payoff in exchange for higher future profit. (This principle is familiar to anyone contemplating law school.) Choice (A) looks like a great choice. The student who turns down a job offer to pursue higher studies definitely exemplifies the principle in the argument. Choice (B) doesn't exactly fit the principle. It discusses the benefits of investing for the future but doesn't mention sacrificing immediate gratification here. Choice (C) is wrong. It recommends a particular action because there's a correlation between that action and future success, but attending church isn't an example of postponing immediate gratification to invest in the future (the author doesn't mention anything that one might be giving up by going to church). Choice (D) is wrong because, although Habitat for Humanity offers a great opportunity for poor people who want to own homes, neither the volunteer builders nor the new homeowners are declining a current opportunity to invest more heavily in an investment with a higher future payoff. Choice (E) may be sound advice, but it doesn't duplicate the argument's principle. In fact, if you buy good furniture now, you're not delaying anything. Choice (A) is the best answer.

18. D. Car manufacturers that design cars according to female tastes can earn larger profits than those who do not.

You want to find the answer best supported by the statements, which note that women buy half of all cars and like to buy vehicles that fulfill their needs and are less harmful to the environment. Look for the answer that incorporates all the statements: Plenty of women buy cars, women notice certain details, and cars designed with those details outsell cars that aren't designed with those details. Choice (A) is wrong because it doesn't mention what women want. You can almost certainly find a better choice than this. Choice (B) is wrong because nothing in the statements allows you to infer the particular type of car that women would like. Choice (C) may be true, but it doesn't address the statements regarding women's vehicle preferences. Choice (D) looks great. It relates women's vehicle preferences to how car manufactures should respond to those preferences, so it's a very good answer. Choice (E) is wrong. Although many people lament the passing of the station wagon, and it's entirely possible that car manufacturers may find that they still have a huge market for these vehicles, the argument isn't concerned with particular types of cars. Choice (D) is your answer.

19. C. children can ever benefit from watching television

The question wants you to identify the source of Carter and Neri's disagreement. Carter says that kids shouldn't watch TV because it gives them attention deficit disorder (ADD). Neri says that a blanket prohibition on children's TV watching isn't necessary because the disorder comes from watching a particular kind of program, and some programs are actually beneficial. Both speakers agree that TV can cause ADD; they disagree on whether TV can ever benefit children. Choice (A) is wrong. Carter and Neri haven't mentioned violence on TV, so you can't possibly know what they think about it. Choice (B) is wrong. Although Carter does mention the significance of spending more time watching TV, he doesn't mention specific durations. Neri doesn't address the amount of TV watching time at all, so you don't know whether they disagree about that. Choice (C) looks right; they definitely disagree about whether children can ever benefit from watching TV. Choice (D) is wrong. Neri does mention educational programs, but Carter doesn't specifically address them, and neither speaker discusses the past generation of TV watchers. Choice (E) is wrong because neither speaker addresses this issue. Choice (C) is correct.

20. D. South Carolina's Congaree Swamp National Monument saw an increase in visitors after it changed its name to Congaree National Park.

The question asks you to support the park ranger's assertion, which means you have to find an answer that provides evidence that the assertion is correct. The park ranger claims that parks and monuments with good names get the most visitors, so the National Park Service should give its parks and monuments appealing names. Look for an answer that shows how giving a park a good name attracts tourists. Choice (A) is totally wrong; it doesn't mention park names at all. Choice (B) is wrong because it attributes attractiveness of parks to location and scenery rather than name. Choice (C) may work because it does compare two parks with different names, but it doesn't say anything about how many people visit either park, so it's not a good choice. Choice (D) looks perfect; a park with more visitors after changing its name provides an example that supports the ranger's conclusion. Choice (E) is wrong. It claims that Yellowstone attracts visitors because people want to see its sights, not because they like the name “Yellowstone.” Choice (D) is correct.

21. E. Recent studies have shown that nonsmokers who are exposed to secondhand smoke suffer more heart attacks, slower-healing wounds, and in general, higher mortality than people who breathe smoke-free air.

You need to find evidence that weakens the argument. The argument concludes that the government shouldn't pass laws outlawing public smoking. The reason for this conclusion is that smoking hurts no one but the smoker, and smokers have the right to choose whether to hurt themselves. What weakens this argument? If the argument is wrong about the effects of smoking on innocent bystanders, then it weakens the contention that smokers hurt only themselves. Look for an answer that mentions the effects of smoking on nonsmokers.

Choice (A) is incorrect; complaints about political inconsistency don't hurt the argument. Choice (B) doesn't weaken the argument at all; you need evidence that smoking affects nonsmokers, not information about what happens to smokers after they stop smoking. Choice (C) is wrong. It does emphasize the damage that smoking does to smokers, but the argument already admitted that and claimed that smokers should be allowed to take on the risk of giving themselves lung cancer. Choice (D) doesn't affect the argument either way; whether or not smoking is decreasing doesn't affect the claim that smokers aren't hurting anyone but themselves. Choice (E) is a perfect answer. The argument bases its conclusion on the claim that smoking doesn't hurt nonsmokers, but this answer completely refutes that claim. If smoking does indeed hurt nonsmokers, the government has a clear interest in protecting them by limiting smoking. Choice (E) is your answer.

22. B. presupposes that actors become extroverted because they study drama

The question asks you to determine where the argument is vulnerable. The argument claims that studying drama and performing can make a person more extroverted; the evidence for this conclusion is that actors are extroverted. The problem with this claim is that you don't know for sure whether actors are extroverted because they studied drama or people who are extroverted to begin with choose drama as a profession. If the latter is true, then studying drama won't make an introvert into an extrovert. Choice (A) is incorrect because the argument makes no judgments about the benefits of extroversion or introversion. Choice (B) looks correct; the argument assumes that acting makes people extroverted and ignores the idea that acting may attract extroverted people. Choice (C) isn't right. The argument doesn't supply much evidence of extroversion, but it does supply a little. However, a lack of evidence isn't the main problem. Choice (D) is wrong because the argument does define “extrovert” somewhat — as someone who is comfortable before a crowd of people. Even if it didn't define the term, that's not the argument's chief weakness. Choice (E) is wrong. The author never denies that there may be other ways for people to achieve extroversion and never suggests that acting is the only way. Choice (B) is right.

23. A. Although a computer-created proof might be too complex to check by hand in the traditional manner, computers are reliable enough today that mathematicians should be willing to admit proofs done by computers into the body of accepted mathematical knowledge.

The question asks you to identify the mathematician's conclusion. This is a long and complex argument on an esoteric topic, but don't let that bug you. Essentially, the mathematician is suggesting that the mathematical community should accept the computer-created proof of the orange-stacking concept; his reason is that computers are very accurate, and all the parts that have been hand-checked so far have been correct. So look for an answer that says this. The validity of the proof is the key. Choice (A) seems to be exactly what you want; the argument does conclude that computers are reliable enough to create acceptable mathematical proofs. Choice (B) is wrong because the mathematician isn't concerned with alternate methods of stacking oranges. He doesn't mention them at all, so they can't be part of his conclusion. Choice (C) is wrong because it's the exact opposite of the mathematician's conclusion; he thinks computers do have a place in the world of theoretical mathematics. Choice (E) is wrong because it describes the process of proving a mathematical theorem, not the mathematician's actual conclusion. Choice (A) is correct.

24. B. Accidents that occur at a higher speed are much more likely to be fatal than those at lower speeds.

The question asks you to resolve the discrepancy that the overall percentage of accidents per drivers hasn't increased since states raised their speed limits, but total accident fatalities have increased. So even though the number of accidents hasn't increased, the accidents that do occur have become more deadly, which is entirely possible. Look for an answer that connects cause (higher speeds) with effect (more fatal accidents despite fewer total accidents). Choice (A) doesn't explain the rise in fatalities because the argument doesn't mention illegal speeding, just higher legal speeds. Choice (B) looks correct. If higher speeds make accidents more deadly, then it makes sense for fatalities to rise even if total accident numbers don't. Choice (C) is wrong because the author isn't interested in resolving a statistical paradox rather than providing suggestions for state regulation. Choice (D) is wrong because it doesn't connect the lower speed limit with fewer fatalities. Perhaps slower cars caused fewer deaths, but the answer doesn't say that. Choice (E) is wrong because it attributes accidents to distractions rather than speed, and it doesn't distinguish between fatal and nonfatal accidents. Choice (B) is the best answer.

25. D. Studies have shown that audiences appreciate the opportunity to laugh while listening to a speech.

You need to find the answer that doesn't weaken the argument; that means eliminating the four answers that do weaken it. The argument's conclusion is that speakers should always start their speeches with a joke, even in foreign countries, because audiences in general like jokes. What would weaken this claim? Anything that disproves the contention that jokes work for foreign audiences. Choice (A) definitely weakens it; speakers shouldn't tell jokes if they'll offend their audiences. Choice (B) weakens the argument, at least as far as female speakers are concerned. Choice (C) weakens the argument; the last thing a speaker needs is for an interpreter to refuse to translate the speech. Choice (D) actually doesn't weaken the argument; if audiences like to relax by laughing, then telling a joke is a good idea. Choice (E) weakens the argument; no speaker wants to inadvertently commit an act of aggression. Choice (D) is the best answer.

Section II: Reading Comprehension

1. E. Nishiwaki was a Japanese poet and scholar who spent his life specializing in European literature, which proved tremendously influential to his own work.

This passage is biographical, telling the life story and listing significant contributions of a major Japanese literary figure. The author is fairly objective and doesn't seem to have much of an agenda beyond emphasizing Nishiwaki's embrace of European literature and culture. (Note, though, that this is an agenda. The author had to choose which details of Nishiwaki's life most interested him and chose to focus on Nishiwaki's interest in Europe. He didn't have to do this; for example, he could have emphasized Nishiwaki's participation in the avant-garde circles of the time or spent more time describing Nishiwaki's work and less on his life story. No text is a foregone conclusion before the author begins to write it.) Choice (A) is wrong. Nishiwaki did protest his government's policies, but rebellion against the government certainly isn't the passage's main point; the author mentions it in only one place. Choice (B) isn't exactly the main idea. Nowhere does the author suggest that Nishiwaki's family disapproved of his studying European literature; in fact, his father was still alive when he went to study a European-influenced style of painting, and you have no reason to assume he objected. He did choose to write in English to give himself more freedom than Japanese allowed, but that wasn't explicitly rebellion. Choice (C) isn't right, either; nowhere does the author say that Nishiwaki tried to convince his students that European literature was better than Japanese literature. Choice (D) is wrong, too; as far as you know, Nishiwaki only chose to write his first work in English and then wrote many others in Japanese. Choice (E) is correct, pretty well summing up the passage.

2. A. scholarly interest in the life and works of a significant literary figure

Skim the first words of the answers, which can help you eliminate a few right away. Choices (D) and (E) are certainly wrong; the passage doesn't include any skepticism or envy. Choice (B) is wrong, too. The author doesn't express surprise at Nishiwaki's use of a foreign language; nor does he seem particularly admiring, which eliminates Choice (C). Choice (A) looks like the best choice because this passage is objective; it's definitely scholarly, with all the author's emphasis on literary movements and descriptions of literary works.

3. B. introduce Nishiwaki and his lifelong interest in European culture

The first paragraph informs you that Nishiwaki was a major literary figure who was deeply interested in European culture, and it describes Nishiwaki's early years and education. So look for an answer that says something like that. Choice (A) isn't right. Although the author does mention Nishiwaki's brief study of painting, that doesn't explain the function the paragraph plays within the passage as a whole. Choice (B) looks like the right answer; the first paragraph does introduce Nishiwaki and sets up the reader to expect him to have more contact with European literature. Choice (C) is wrong. Aside from the first sentence, the paragraph doesn't actually say much about Nishiwaki's contribution to Japanese literature. Choice (D) isn't right, and in fact, nowhere in the passage does the author attempt to explain Nishiwaki's choice beyond noting that Nishiwaki found European literature interesting. Choice (E) is clearly wrong; this paragraph has nothing to do with European contributions to Japanese culture. Choice (B) is correct.

4. C. He disapproved of the Japanese government's policies and in protest refused to write poetry.

The relevant sentence reads, “Angered by the Japanese government's fascist policies, Nishiwaki refused to write poetry during the second world war.” That makes Choices (A), (B), and (D) obviously wrong. Choice (C) rephrases that sentence, which makes it look like a good answer. Choice (E) may confuse you because the following sentence does tell you that Nishiwaki spent the war years writing a dissertation on German literature, but the way the two sentences are phrased makes it clear that the dissertation was simply how Nishiwaki chose to fill his time because he had already made the decision not to write poetry. So Choice (C) is correct.

5. C. providing a brief biography of Nishiwaki that explains the significance of his work

This is a main idea question. What's the main idea of this passage? To provide a biography of a Japanese writer, explaining why his work is important. Skim the answer choices and see if one of them matches. Sure enough, Choice (C) looks like a good answer. Check the others just in case. Choice (A) is wrong because the author doesn't mention the work of other Japanese poets. Choice (B) is wrong because although the avant-garde movement did influence Nishiwaki's writing, the author's discussion of it takes up only a small part of the entire passage. Choice (D) is wrong because the author doesn't mention the benefits of studying foreign literature. Choice (E) is wrong because the passage doesn't discuss changes in overall Japanese poetic style after the war. Choice (C) is the best answer.

6. E. classical Japanese literature such as The Tale of Genji

You know that Nishiwaki studied Old and Middle English, classical Greek and Latin, and a smattering of modernist English works at Oxford. That covers Choices (A) through (D) and leaves only Choice (E). You also know that Nishiwaki didn't especially love traditional Japanese poetic forms, of which The Tale of Genji is an example. That makes Choice (E) the best answer. (Yes, Nishiwaki did have some interest in Japanese literature, and he may well have appreciated The Tale of Genji, but the passage provides no basis for thinking that he actively pursued an interest in classical Japanese literature.)

7. A. He found that English allowed him to express a wider variety of thoughts and emotions than Japanese.

Here's the relevant sentence: “In 1925, Nishiwaki published his first book, Spectrum, a volume of poems written in English; he explained that English offered him much more freedom of expression than traditional Japanese poetic language.” That makes Choice (A) the only sensible answer. The rest of them are just plain wrong.

8. A. Events of the last two decades have shown that sea urchins play a vital role in the maintenance of the ecosystems in which they live, contrary to what many scientists formerly assumed.

This passage is about the importance of sea urchins to the marine environment, which has come as a surprise to many people who formerly discounted the significance of the spiny creatures. Choice (A) sums this up pretty well; it looks like a good answer. Choice (B) isn't really the main point; the author mentions a project to relocate sea urchins in the last paragraph, but that doesn't play a major role in the passage as a whole. Choice (C) isn't the main point, though it may be true; more important, the passage is about sea urchins, not all echinoderms. Choice (D) is also true, but it's not the main point either. The author uses information about excessive algae growth to prove her point that sea urchins are now recognized as important to the ecosystem. Choice (E) is just plain wrong; nowhere does the author mention the government's duty to educate people about the environment. Choice (A) is the correct answer.

9. B. describe the appearance and eating habits of sea urchins

The second paragraph is a concise description of urchins and their activities. Skim the first words of the answer choices to see if one of them uses a word like “describe.” Choice (B) does, and it looks like a good answer. Quickly check the others to make sure Choice (B) is best. Choice (A) is wrong because the second paragraph doesn't claim that urchins are important to the ecosystem; that claim is elsewhere. Choice (C) is wrong because its claim is also in another part of the passage, not in the second paragraph. Choice (D) is wrong because that information is in the first and last paragraphs. Choice (E) is wrong because the author doesn't mention the diets of other echinoderms. So Choice (B) really is the best answer.

10. C. They are all members of the same phylum.

The second paragraph says: “Sea urchins are members of the phylum Echinodermata, the category of invertebrate animals with spiny exterior shells, which also includes starfish, brittle stars, and sand dollars.” So all four are members of the same phylum. Is there an answer that says that? Yes — Choice (C) looks like your answer. The rest of the information in this paragraph regards only sea urchins and doesn't apply to the other species. Choice (A) is wrong because although the passage indicates that echinoderms are invertebrates, it doesn't say that all echinoderms eat algae; you only know that sea urchins eat algae. As far as you know, only sea urchins were devastated by the bacterium in 1983, so Choice (B) is wrong. Choice (D) is a bit tricky because the second paragraph does say that all echinoderms have spiny shells, but it doesn't say that all the spines are long and sharp. Choice (E) happens to be true of echinoderms, but the passage doesn't tell you that. So Choice (C) is the best answer.

Note: This passage doesn't really require you to know much about invertebrate taxonomy, but the LSAT-makers do assume that you remember some rudiments from your science classes, such as the meaning of the word phylum. (You don't have to know a very specific meaning, but you should remember that it's a kind of category into which scientists sort animals and plants based on their physical characteristics.)

11. E. A bacterium carried by a ship entering the Atlantic from the Pacific spread throughout the population and killed nearly all sea urchins.

Refer to this quotation: “In 1983 a barge traveled from the Pacific to the Atlantic through the Panama Canal carrying a bacterium that proved devastating to the sea urchin population. Nearly all long-spined urchins in the tropical western Atlantic died as a result of this exposure.” Choice (E) is the only answer that sums up this tragedy. The other answers are simply wrong. The author does mention people killing sea urchins but doesn't connect this with their near eradication, so Choice (A) is wrong. Choice (B) is wrong because the passage says that coral is dying because it's covered with algae, not that it was killed by a plague. Nowhere does the author mention triggerfish, so Choice (C) is wrong. Choice (D) isn't right either, because a bacterium killed the urchins, not a virus (they're very different), and it didn't come from other echinoderms. That makes Choice (E) the answer.

12. C. Coral reefs in regions that still have healthy sea urchin populations have almost no algae growing on them.

What would support the claim that sea urchins are key to solving the algae on reefs problem? Well, if someone had examined reefs with and without sea urchins, and the reefs without urchins had algae overgrowth problems and those with urchins didn't, all other conditions being equal, that would tend to support this hypothesis. Choice (C) looks like the best answer. Choice (A) doesn't work because knowing that damselfish could also take care of the algae problem would actually weaken the author's claim that sea urchins are key to algae removal from coral reefs. Choice (B) is irrelevant because the issue is algae elimination, not coral elimination. Choice (D) has no effect on the author's claim because the article never differentiates among different types of algae. Choice (E) is interesting, but the passage doesn't distinguish types of coral, so it's not clear how this point about coral would affect the claim that sea urchins will take care of the algae problem. Choice (C) is by far the best answer.

13. C. provide an example of how the scientific community has altered its view of sea urchins

To answer a question about the purpose of a part of the passage, you need to view it in a larger context, which often means reading at least a sentence or two before and after the part the question references. The Environmental Protection Agency (EPA) is one of two examples showing scientists’ changing views on the importance of sea urchins. Choice (A) is wrong because, though it describes the EPA's actions, it doesn't explain that the reason the author mentions those actions is to provide an example of scientists’ changing views. Choice (B) is wrong; nowhere does the author mention any relationship between government and shipping. Choice (C) addresses the inclusion of the EPA as an example and therefore looks like a potential right answer. Choice (D) is wrong and, in fact, backward, because EPA scientists have recognized the significance of sea urchins. Choice (E) is also wrong because nowhere does the author mention voters. Choice (C) is the correct answer.

14. A. Humans can take an active role in assisting the recovery of ecosystems that have been harmed by human actions.

The correct answer is probably a statement that recaps the author's main point or one of her items of evidence. Choice (A) looks like something the author would say. In the last paragraph, she writes approvingly of the scientists who are currently relocating sea urchins to algae-covered reefs. Choice (B) contradicts Choice (A); if the author likes the statement in Choice (A), she won't agree with the one in Choice (B). Choice (C) is just plain wrong; in the first paragraph, the author writes of the ignorant folks who used to kill sea urchins for sports, not realizing the importance of their victims. Choice (D) may well be true in the real world, but you can't really tell from this passage because the author never mentions punishment for the shipping company that introduced the deadly bacterium; Choice (A) still looks better. Choice (E) is almost certainly not something the author would say. Based on her evidence, no creature has yet adapted itself to the sea urchins’ niche, and that's why corals are now covered with algae. Choice (A) is the best answer.

15. D. nonprofits may face difficulties staying true to their overall mission

The two passages discuss potential problems in running a nonprofit. Passage A concentrates on the difficulties some nonprofits have with leadership and finances. Passage B's focus is on the potential for fraud and corruption in nonprofits.

Choices (A) and (C) involve concerns cited by the author of Passage A but not B, and only Passage B mentions Choice (B). Only Passage B suggests that board members may put their own financial interests above the mission of the nonprofit, so Choice (E) isn't correct. Both authors discuss Choice (D).

The author of Passage A notes that one of the most important challenges is “. . . to keep the mission in mind in all decision making,” and Passage B's author notes that one of the primary responsibilities of a nonprofit's governing board is to “make sure the organization remains true to its mission.”

16. A. Managing the Nonprofit Sector: An Overview

Questions that ask for a title are main idea questions, and the best answer to a main idea question is the one that's neither too general nor too specific. Eliminate answers that concern only one of the passages and select the one that is general enough to pertain to both.

Passage A briefly discusses Choice (B) when the author discusses the decline in government support and the growing number of for-profit businesses occupying areas once traditionally occupied by nonprofit organizations, but it's not the main idea and isn't even mentioned by Passage B. Both passages briefly touch on the conflict between nonprofits’ missions and funders’ missions, but that conflict isn't the main idea of either passage, so Choice (C) is too specific. If you picked Choice (D), the last line of Passage B is probably still fresh in your mind, but while both passages discuss the role of the board in managing nonprofits, they also consider other factors. You may wish to keep Choice (D) in mind as you look for a better option. Choice (E) implies that the passages focus on the nonprofit sector and how it has changed and advanced over time, and with the exception of a reference here or there as to how things have been done in the past, neither passage focuses on the history of nonprofits. Choice (A) is a bit vague but at least summarizes the overall purpose of both passages to discuss the management challenges experienced by nonprofits. It's more on point than Choice (E) and less issue-specific than the other choices, making Choice (A) the best choice of those available. Remember, sometimes the correct answer will be the best one out of five lousy choices.

17. E. Nonprofits lack a strong defense against corruption and fraud.

Choice (E) is the only option listed that isn't backed up by the information contained in Passage B. In fact, the opposite is true — the author of the passage states that there are in fact “many lines of defense against fraud and corruption.” All others are mentioned in Passage B. Choices (A) and (C) are paraphrases of statements in the third paragraph, Choice (B) is the information in parentheses in the last paragraph, and Choice (D) appears in the second paragraph.

18. D. The public should hold managers, leaders, and boards of nonprofit organizations liable for their decisions.

Choice (C) is an assertion made by only Passage A, so there's no way to know whether the author of Passage B would agree. The same goes for Choices (A), (B), and (E). So you've already narrowed down your answer to Choice (D). Because Choice (D) also sounds like something relevant to only Passage A, spotting the wrong answers to this question may be easier than finding the right answer. Note that the author of Passage B states that a nonprofit board is “responsible for making sure the organization remains true to its mission, safeguards it assets and operates in the public interest.” Similarly, the author of Passage A notes that “the real owner of a nonprofit is the public. It is the public to whom they are ultimately accountable.” So you can safely pick Choice (D).

19. B. The nonprofit sector does not benefit from oversight by a “watchdog” organization whose purpose is to promote integrity, as businesses and the government do.

Choice (B) is essentially a paraphrase of the entire second paragraph of Passage B, so it's clearly something indicated by the passage. If you picked Choice (A), you misread the first line of Passage B, which actually says that nonprofits are not immune from any damage that could be inflicted by unscrupulous or fraudulent solicitors. Choices (C) and (D) are statements included in Passage A rather than Passage B, so nix those options as well. As for Choice (E), while Passage B does indeed mention the media, it does so in reference to the fact that they monitor business and government, not nonprofits. Stick with Choice (B).

20. C. Most people do not understand how nonprofit organizations are regulated and monitored.

If you didn't pick Choice (C), you may have forgotten the general rule that you must rely solely on the information contained in the passage, regardless of what your own personal knowledge (or common sense) tells you. While both authors may theoretically agree with Choice (C), it's only brought up in Passage B, so you can't make the assumption that both authors would agree.

Choice (A) is stated in one way or another by both passages; Passage A states that “The real owner of a nonprofit is the public,” and Passage B says that “All nonprofits are governed by a board of directors or trustees.” Both imply that more than one individual is leading the show. Choice (B), while vague, conveys a central point from both passages and is, to some degree, a paraphrase of the information contained in each. So it can't be right. Choice (D) is also an underlying theme present in both passages, so you can knock that one out of contention as well. Regarding Choice (E), Passage A refers to pressure from funders in its final line when it says that funders demand to see positive, measurable results stem from the money they contribute. Passage B references pressure from stakeholders when it says that nonprofits are threatened by board members who are more concerned with their personal financial well-being than the overall mission. Stick with Choice (C).

21. E. indicate the role that government plays in the operation of nonprofits

Focus on the context in which each passage mentions the government. Passage A brings up the government to show how the decrease in government funding has negatively affected the well-being of nonprofits. Passage B talks about the government to emphasize that the government doesn't do much to regulate nonprofits. Both show that the government has some role in relation to nonprofits, so Choice (A) is out. Choice (B) can't be right because the second paragraph of Passage A lists “all levels of government” as one of the stakeholders in nonprofits. Choice (C) relates to only Passage A, so it doesn't work. Neither passage suggests that nonprofits and government agencies are alike, so Choice (D) is wrong. Whenever the passages mention the government, they provide one of the ways the government does or doesn't influence the running of nonprofits. So Choice (E) is the best answer.

22. D. Cloning: Just Another Way of Making a Baby

A title question is similar to a main idea question. To come up with an accurate title, you have to understand what the whole passage is about. In this case, what is the author's point? She's arguing that the ethical debates about cloning don't make sense because cloning is more similar than dissimilar to regular reproduction and faces many of the same issues.

Choice (A) doesn't make a very good title. The author is explicitly arguing that cloning would not harm families. Choice (B) is wrong because this article isn't about the dangers of cloning. Although it does mention several dangers that people have suggested, both ethical and physical, they're not the point. Choice (C) isn't right; the author briefly discusses the mechanics of cloning in the first paragraph but by way of introducing the subject, not as a thesis. Choice (D) makes a perfect title; that cloning is just another way of conceiving a child is exactly the author's point. Choice (E) doesn't work because the passage isn't about the government's role in the debate or about emphasizing the unethical aspects of cloning. In fact, aside from a quick reference to the Clinton administration in the first paragraph and a brief mention of the government's lack of interference in reproduction issues, the government doesn't play a role in this passage. Choice (D) looks like the right answer.

23. D. Most critics of cloning are worried about aspects of cloning that are not substantially different from the potential problems associated with methods of reproduction already in common use.

The author's point is that the criticisms of cloning are aimed at fanciful concerns and fantasies about “natural” relationships between parents and children. She would probably think that the people who have criticized cloning based on these concerns are arguing from emotion, not fact, and that their arguments are flawed because they could potentially apply to normal reproduction as well. Choice (A) doesn't work; the author doesn't ever suggest that cloning critics are religious fanatics. Choice (B) is definitely wrong; the author raises the point about creating a master race as an illustration of an outrageous fear so unlikely as to be laughable. Choice (C) is wrong; though the author doesn't state this explicitly, she implies that foes of cloning don't object to other techniques of assisted reproduction. Choice (D) does match up with the author's point; she would argue that children already enter the world in difficult and unpredictable circumstances and that parents already treat children differently depending on their expectations even when they conceive their kids the usual way. Choice (E) is clearly wrong; the author says that cloning critics tend to ignore these practical considerations in favor of fanciful ones. Choice (D) is correct.

24. A. argue that most critics of cloning are focusing on drawbacks that are either outlandish or no different from the drawbacks of normal reproduction and that ethically cloning is not very different from normal reproduction

Based on the main idea of the passage, Choice (A) looks like a very good answer. The author is arguing that cloning critics have seized on concerns that aren't really valid and that cloning shouldn't be all that different from other techniques of conception. Choice (B) isn't right because most of the passage isn't concerned with the mechanics of cloning or other genetic engineering processes. Choice (C) doesn't work; the author doesn't address scientists particularly and makes no explicit recommendations. Choice (D) looks like it might work because the author does list some potential benefits of cloning, but it's really not her main purpose. So Choice (A) still looks like the best answer. Choice (E) is definitely wrong; if anything, the author is doing the opposite, suggesting that many so-called dangers of cloning are nothing of the kind. Choice (A) is the best choice.

25. E. show that the worries of cloning critics are exaggerated and based more in fantasy than fact

“Nefarious” and “abominable” are very strong words, commonly used to describe only the most evil of people and deeds. They're definitely too strong to describe the personalities and intentions of most scientists. The author uses them here in a tongue-in-cheek fashion to emphasize the fantastic nature of many criticisms of cloning and to suggest that these fears are probably derived from science fiction rather than real information. Note, too, that these two words are an aberration from the writer's usual vocabulary, which is generally down-to-earth. The best answer is Choice (E).

26. D. In vitro fertilization is a sensible and positive use of technology to help people accomplish a natural human goal.

Based on her comments, the author seems to believe assisted reproduction is a perfectly acceptable way to have children. She wouldn't agree with Choice (A); her statements in the last paragraphs show that she doesn't find much difference in the results of various reproductive techniques. Choice (B) is definitely wrong. Though the author doesn't explicitly state that the government should regulate according to ethics, she implies that the benefits of cloning outweigh the risks and that the government ban on cloning research is misplaced. She would almost certainly extend this opinion to regulation of other types of reproductive technology. Choice (C) is wrong because the author's point is that, although an imbalance in the way parents treat children conceived through assisted reproduction is a hypothetical risk, in fact, regardless of how they're conceived, all kids face parental expectations. And in any case, placing higher expectations on a child doesn't mean a parent values that child more highly. Choice (D) looks good; the author obviously thinks reproductive techniques should be allowed and that they're not all that different from normal conception. The author wouldn't agree with Choice (E); among the benefits of cloning, she includes helping same-sex or infertile couples reproduce, and if she would allow it for cloning, she would allow it for other forms of assisted reproduction. Choice (D) is the best answer.

27. E. Cloned embryos have a high risk of dying during gestation and before birth.

To answer this question, you have to look back at the passage and find sentences that mention the various risks of cloning. Choices (A) and (B) appear at the end of the third paragraph. Choices (C) and (D) both appear in the last paragraph. The author mentions these as risks that she doesn't think are really risks, but she does mention them in the passage, so they can't be right. Choice (E) is the only risk that the author doesn't mention, so Choice (E) is correct.

Section III: Analytical Reasoning

Questions 1–6

To create a game board for this grouping set of questions, consider the fact that you have two types of game pieces: instructors and types of yoga. First, abbreviate the instructors by initials: C, J, M, S, and V. Abbreviate the kinds of yoga as a, b, and i. You're grouping instructors by the type of yoga they teach, and anywhere from one to three instructors teach each type of yoga. Keep track of this knowledge on your game board by providing a maximum of three spaces for each of the three types of yoga.

Write the rules in shorthand under the game board. Two and only two instructors must teach the same class or classes. M and S both teach iyengar, so add them under “i” on the game board. For the next rule, write C < J and M < J to designate that C and M teach fewer kinds of yoga than J. For the last two rules, write V = no C and its contrapositive, that C = no V, and write J = no V and its contrapositive, V = no J.

Now consider the ramifications of the rules. If C and M both teach fewer classes than J, then they must teach either one or two classes, and J must teach either two or three. But you know that J doesn't teach any classes that V teaches, which means J can't teach three classes and must teach two. Therefore, C and M must each teach one class. Record these rules on your game board. You already know that M teaches iyengar, so M never teaches ashtanga or bikram. You also know that V can teach only one class, because she doesn't overlap with J's two classes. What about S? Well, at this point it looks like she could teach one, two, or three classes; you can't decide. Mark these numbers on the diagram using either blank spaces or numbers, or both. Here's what your game board may look like:

9781118678237-un20001.tif

1. D. Virginia teaches fewer kinds of yoga than Janice does.

You can answer this question by eliminating answers that must be false or could be true. Your game board reveals that Caroline and Virginia each teach one kind of yoga, so Choice (A) is wrong. You don't know how many kinds of yoga Suzanne teaches, so Choice (B) doesn't have to be true, though it could be. Suzanne might teach fewer kinds of yoga than Janice, but she doesn't have to, so Choice (C) is wrong. You know that Virginia can teach only one kind of yoga, and Janice teaches two, so Choice (D) looks right. Double-checking Choice (E), Virginia may or may not teach fewer kinds of yoga than Suzanne, so Choice (E) is wrong. Choice (D) it is.

2. B. Janice teaches iyengar.

This question adds a temporary rule. If Virginia doesn't teach iyengar, she must teach either ashtanga or bikram. You know that Janice teaches the classes Virginia doesn't teach, which means Janice must teach iyengar. When you record these new conditions on your game board, it looks like this:

9781118678237-un20002.tif

It looks like Choice (B) should be the answer. But you can check the other answers just in case. Janice will teach either bikram or ashtanga, whichever one Virginia doesn't teach, but you can't know which one from the given information; that knocks out both Choices (A) and (E). Marty can teach only iyengar, which eliminates Choice (C). Suzanne may be able to teach ashtanga, but she doesn't have to, which knocks out Choice (D). Choice (B) is correct.

3. A. Caroline

Can you eliminate any choices? You know Janice teaches two classes, so any choice including her can't be right; cross off Choice (B). You know Caroline and Virginia can't teach the same class, so cross off Choices (C) and (E). You know Marty teaches only iyengar, so cross off Choice (D). Could Caroline teach only bikram? Yes, she could teach only one kind of yoga, and it could be bikram. So the answer must be Choice (A).

4. E. Janice, Marty, Suzanne

You know Marty and Suzanne must teach iyengar, so you can eliminate any choice that doesn't include both of them; cross off Choices (B) and (C). Consider Choice (A) a possible right answer, but test the other choices to see if the list could be expanded; remember, you want a complete list. Could Caroline teach iyengar? No, if Caroline teaches iyengar, Virginia can't teach iyengar, but if Virginia can't teach iyengar, then Janice must teach iyengar. Choice (D) doesn't include Janice, so it's not a complete list. How about Choice (E)? Sure; if Janice teaches iyengar, she can also teach ashtanga, Caroline could teach ashtanga, too, which would leave bikram for Virginia. If Suzanne also teaches ashtanga, Suzanne and Janice satisfy the condition that two instructors teach the same yoga type. The grouping looks like this:

9781118678237-un20003.tif

The answer is Choice (E).

5. C. three

Caroline, Marty, and Virginia must teach only one kind of yoga. Janice must teach two kinds, and you don't know about Suzanne. She could teach only one but could also teach two or three, so you can't say she must teach only one kind. The answer is three, Choice (C).

Tip: Pay attention to “must,” “can” and “could,” and “all/except” questions. Underline these words and think about what the right and wrong answers will mean before you begin working out the question. In this case, if only one “must” be true, all the other choices are merely possible or false. By thinking this through beforehand, you're less likely to jump at the first possible choice.

6. D. Suzanne teaches bikram.

Consider the temporary new rule offered by this question. The three instructors who teach ashtanga can't include Marty because she teaches only iyengar. That eliminates Choice (C). Can Caroline teach iyengar? If Caroline teaches iyengar, the three instructors that teach ashtanga have to be Suzanne, Virginia, and Janice, but Virginia can't teach the same type of yoga as Janice. Choice (A) can't be true. Can Suzanne teach bikram? If Suzanne teaches bikram, she can also be one of the three ashram teachers. The other two have to be Caroline and Janice because Virginia can't teach the same type of yoga as either Caroline or Janice. Virginia then must teach bikram with Suzanne, and Janice must teach iyengar with Marty and Suzanne to fall in line with the first rule that two instructors teach the same type of yoga. The grouping looks like this:

9781118678237-un20004.tif

Choice (D) is possible, so it must be correct. When you consider Choices (B) and (E), you realize that the three ashtanga instructors have to be Suzanne, Janice, and Caroline to keep Virginia separate from both Caroline and Janice. So Choices (B) and (E) can't be right and the answer is Choice (D).

Questions 7–12

The first step in solving this grouping question set is to abbreviate all the names as initials. So you have A, B, C, D, O, S, and V. Use lowercase letters to mark Patriots and Liberators (p and l). You're going to be sorting these people into two categories, so write two headings — p and l — on your game board and record members under them.

The most important step is to explore the rules. Write them down in shorthand on your game board. Knowing that O and C don't join the same group is the same as saying that O and C occupy different groups, so when O = p, C = l and vice versa. The same is true for A and D. They can't both be p or l together. That also means that there must be a minimum of two and a maximum of five noblemen in each group. The third condition is that if D = p, S = p. The contrapositive is also true: if S = l, D = l. The fourth rule is if B = p, C and V = l, and its contrapositive is if C or V ≠ l, B = l, because whenever at least one of either C or V is p, they can't both be l. When you record the final rule that if O = p, S = l, you see an intersection with the first rule. You can conclude that when O = p, both C and S = l. The second and third rules intersect as well. If A = l, D = p and S = p. The game board looks something like this:

9781118678237-un20005.tif

7. B. Brutus joins the Liberators.

Add the temporary new rule provided by this question to the game board. Jot O down under the Liberators. That automatically puts C under the Patriots. Look at your list of rules; if C ≠ l, then B = l, so put B under the Liberators. Choice (B) must be the answer.

You can check the other answers to be sure. If Antonius joins the Liberators, Decimus and Servilius must be Patriots, and Vipsanius can be either a Patriot or Liberator. Choice (E) is wrong. If Antonius joins the Patriots, Decimus is a Liberator, so Choice (C) is wrong. Servilius or Vipsanius can be either Patriots or Liberators, so Choices (A) and (D) are wrong. Possible groupings look like this:

9781118678237-un20006.tif

The answer is Choice (B).

8. E. Servilius joins the Patriots.

According to the temporary rule posed by the questions, plug the game pieces into the appropriate groups. Eliminate answers that could or must be true. If Cassius is a Liberator, then Octavius is a Patriot. If Octavius is a Patriot, then Servilius is a Liberator. If Servilius is a Liberator, Decimus is a Liberator and Antonius is a Patriot. That leaves Brutus undecided; he could go either way. The groups look like this:

9781118678237-un20007.tif

Skimming the choices, the only one that doesn't match up is Choice (E); Servilius must be a Liberator, so that's the only false statement.

9. A. Brutus and Cassius both join the Liberators.

Jot down A and V under Patriots. Place the other men you know — D goes under Liberators, and so does B because V is under Patriots. Now consider your options, eliminating rule violators first. Choice (B) can go because Brutus isn't a Patriot. Choices (C) and (E) can go because Decimus can't be a Patriot. Choice (D) is definitely wrong because Cassius and Octavius can't be on the same side, which leaves Choice (A). Try Cassius as a liberator; that makes Octavius a Patriot, which makes Servilius a Liberator, like this:

9781118678237-un20008.tif

Choice (A) looks good.

10. B. Antonius and Servilius

Approach this question by applying the rules to each answer one at a time, until you've placed everyone. Put A and O in the Liberator group. If Antonius is a Liberator, then Decimus is a Patriot. If Decimus is a Patriot, then Servilius is a Patriot. If Octavius is a Liberator, then Cassius is a Patriot. If Cassius is a Patriot, then Brutus is a Liberator, and Vipsanius could go either way. That works, so Choice (A) is possible. Your groups look like this:

9781118678237-un20009.tif

Try Choice (B), A and S as Liberators. If Antonius is a Liberator, then Decimus must be a Patriot, which means Servilius must be a Patriot, so Antonius and Servilius can't both be Liberators. Choice (B) is the answer. If you're confident, go ahead and leave it at that; if you have time, work through the rest of the possibilities to be sure you haven't missed something.

Choice (C) gives you this roster of Liberators: Antonius, Vipsanius, Octavius, and Brutus. It gives you these Patriots: Decimus, Servilius, and Cassius. How? If Antonius is a Liberator, Decimus must be a Patriot, which makes Servilius a Patriot, which makes Octavius a Liberator, which makes Cassius a Patriot, which makes Brutus a Liberator. Choice (D) gives you these Liberators: Cassius, Vipsanius, Servilius, and Decimus. It gives you these Patriots: Octavius, Antonius, and Brutus. How? If Cassius is a Liberator, then Octavius is a Patriot, which makes Servilius a Liberator, Decimus a Liberator, and Antonius a Patriot. Cassius and Vipsanius are both Liberators, which makes Brutus a Patriot, too. Choice (E) gives you this roster of Liberators: Octavius, Servilius, Decimus, and Brutus. It gives you these Patriots: Cassius and Antonius. Vipsanius could be either. If Octavius is a Liberator, Cassius is a Patriot, which makes Brutus a Liberator. If Servilius is a Liberator, Decimus must be a Liberator. Vipsanius could go on either side.

The possible grouping for the last three answer choices are these:

9781118678237-un20010.tif

11. D. four

Based on work in previous questions, you can immediately eliminate Choices (A) and (B); you already know you can have at least three Patriots. See if you can concoct a roster with five Patriots (because you're looking for a maximum, and five is the largest number the answer choices offer). Try making Decimus and Servilius Patriots, which forces Antonius to be a Liberator. Brutus isn't a good choice for a Patriot, because he forces Cassius and Vipsanius over to the Liberators, so instead make Cassius and Vipsanius Patriots and let Brutus be a Liberator. That makes Octavius a Liberator, too, because he doesn't play on the same team as Cassius. There doesn't seem to be any way to get five guys onto the Patriots, but you know you can do four. That makes the answer Choice (D).

12. A. Antonius, Cassius, Vipsanius

Look for obvious rule violations first. Antonius and Decimus can't be on the same side, which nixes Choice (B). Making Servilius a Patriot forces Octavius to be a Liberator, which nixes Choice (C). Now try the others. For Choice (A), if Antonius is a Patriot, then Decimus is a Liberator. If Cassius is a Patriot, then Octavius and Brutus are both Liberators. You can make Servilius a Liberator without violating any rules, so Choice (A) is a good choice. Now try Choice (D). If Decimus is a Patriot, then Servilius must also be a Patriot. Servilius isn't in that list, so Choice (D) is wrong. Now for Choice (E): If Decimus is a Patriot, then Antonius must be a Liberator. If Servilius is a Patriot, then Octavius must be a Liberator. If Vipsanius is a Patriot, then Brutus must be a Liberator. Cassius can't be in Octavius’s group, so Cassius must be a Patriot, so the list in Choice (E) isn't complete. The answer is Choice (A).

Questions 13–18

This ordering problem is pretty straightforward, but it can be tricky if you don't sketch it out correctly. As always, start your game board by abbreviating your game pieces: A, C, E, F, H, and O. You have to put the plays in order by days of the week, so make the days the foundation of the game board: Write T, W, R, F, Sa, Su across the page, leaving space underneath to fill in information. You know H must be performed on Thursday, so record that in the Thursday column.

Now consider the other rules. If A comes after O, A doesn't play on Tuesday and O doesn't play on Sunday. Record this information on your game board, and write the rule underneath as O → A. The third rule states that at least one play comes between A and F. It doesn't say that A must come before F. F could come before A, but there will still be at least a one-day break between them. You can record this rule as “no AF or FA” on the game board. The last rule is similar. You know that C and F don't play consecutively, but you also know that exactly one play separates their performances. You can record this rule as C_F or F_C.

Take a look at your game board. Because H is scheduled for Thursday, you know neither C nor F can be performed on Tuesday or Saturday. So Tuesday's only options are E and O and Saturday's performance can be only A, E, or O. Record this information on your game board, like so:

9781118678237-un20011.tif

13. D. Orestes, Frogs, Helen, Clouds, Antigone, Electra

Answering the first question in the set is usually as simple as working through the rules to find which option doesn't create a violation. Check answers for violations of the first rule. Which choice has Helen on a day other than Thursday? Choice (E) does, so you can cross it off. Consider the second rule. Choice (A) has Orestes perform before Antigone, so it's wrong. Does any choice put Antigone and Frogs next to each other? Choice (B) does, so eliminate it. Choice (C) violates the final rule by failing to separate Clouds and Frogs by exactly one day. That leaves Choice (D), which observes all the rules and is therefore correct.

14. D. Either Clouds or Frogs is performed on Friday.

The question asks for what must be true, so eliminate answers that could be true or must be false. The play with the most restrictions is Frogs, so first consider the two choices that include Frogs. Clouds and Frogs can be performed only on Wednesday, Friday, or Sunday. There must be one and only one day between them. The only way to accomplish that is to perform either Clouds or Frogs on Wednesday and Friday or on Friday and Sunday. Either way, you have to perform either Clouds or Frogs on Friday. You can add this information to your game board for future questions. That makes the answer Choice (D).

15. E. Orestes

This question is pretty simple. What play must be performed before another play? Orestes must come before Antigone, which means that Orestes can never be performed on Sunday. The answer is Choice (E).

16. C. Electra and Orestes are performed on consecutive days, with no other play between them.

Sketch out the possibilities presented by the answer choices on your game board. Eliminate those that could or must be true. Try Choice (A). If you put Orestes on Tuesday and Antigone the next day, Wednesday, there's room for Helen on Thursday, Clouds on Friday, Electra on Saturday, and Frogs on Sunday. Choice (A) is possible and therefore incorrect. Try Choice (B). The ordering possibility that worked for Choice (A) also works for Choice (B), so it can't be right either. Consider Choice (C). Friday is reserved for Clouds or Frogs, so it's off limits, as is Thursday. You could put Electra and Orestes on Tuesday and Wednesday, Frogs on Friday, and Clouds on Sunday, but that would force Antigone next to Frogs, which is forbidden. If you put Electra and Orestes on Saturday and Sunday, that would place Orestes after Antigone, which is also a rule violation. Choice (C) must be false. Just to make sure you haven't missed something, you may wish to consider the other possibilities. You know Choice (E) is possible from the ordering you used for Choices (A) and (B). If you put Orestes on Tuesday and Frogs next to it on Wednesday, there's space for Helen on Thursday, Clouds on Friday, and Antigone and Electra on Saturday and Sunday. Choices (D) and (E) are possible, so they can't be right. The possible orders for Choices (A), (B), (D), and (E) could look like this:

9781118678237-un20012.tif

The answer is Choice (C).

17. D. Wednesday, Friday, Sunday

You know from your game board that the possible performance dates for Frogs are Wednesday, Friday, and Sunday, so you can eliminate any choice that has a day other than those three: Choices (A), (B), and (E). Double-check Sunday to make sure it really can work for Frogs. If you put Frogs on Sunday, Clouds plays on Friday. To avoid scheduling Antigone next to Frogs, put Orestes on Tuesday and Antigone on Wednesday, which leaves Thursday open for Helen and Saturday available for Electra. Frogs can definitely play on Sunday. The answer is Choice (D).

18. B. Electra is performed on Wednesday.

Based on your answers to other questions, you know that Frogs and Clouds can be performed on Wednesday, so cross off Choices (A) and (C). Try Choice (B). If Electra is performed on Wednesday, then Clouds and Frogsmust be performed on Friday and Sunday. But you run into a problem. With Helen on Thursday, only Tuesday and Saturday remain for the performances of Orestes and Antigone. If you put Orestes on Tuesday, Antigone gets stuck next to Frogs on Saturday. If you put Antigone in the Tuesday spot, Orestes improperly performs after Antigone. Either way you violate a rule, as shown in this potential ordering:

9781118678237-un20013.tif

Choice (B) is your answer. The last two choices work and therefore can't be right. Put O on Tuesday and several acceptable schedules result, such as O, F, H, C, A, E or O, A, H, C, E, F. With O performed on Saturday, you could have the schedule look like this: E, C, H, F, O, A. The answer is Choice (B).

Questions 19–25

This question set involves grouping attorneys with specialties. Because there will always be only four specialties, and attorneys have multiple specialties, this grouping problem is the more complex, open type.

Begin arranging your game board by listing the game pieces. There are four attorneys: B, C, D, and H. The four possible areas of law are e, i, r, and t.

At most each attorney specializes in three of the four areas. As you read the rules, notice that they specify exactly how many attorneys practice each specialization. This tells you that it may be easier to set up your game board with the spots for attorneys listed under each specialty. Put one spot under tax, two under employment and real estate, and three under immigration.

Now consider the other two conditions. The first rule is the same as saying “if C then D.” The contrapositive is also true: If no D, then no C. It does not mean that if D specializes in the area of law, then C does. The other rule is that “if C, no B and no H.” The contrapositive is “if B and H, then no C.” Write these rule shortcuts on your game board so that it looks something like this:

9781118678237-un20014.tif

You may come up with a few other deductions that will make your thinking process easier as you answer questions about this problem. First, if D must share C's specialties, then C is never the sole specialist in an area. That means C can't specialize in tax. Second, if C can't share a specialty with B or H, that means the only three attorneys who can specialize in the same area must be B, H, and D. They must be the three who specialize in immigration. Third, C must be paired with D in any specialty, and only the two of them can specialize in that particular area, which means that they must practice either employment or real estate law, because those areas need exactly two attorneys. This information is valuable; note it on your game board:

9781118678237-un20015.tif

19. C. Duway does not specialize in tax law.

Eliminate answers that could be true or must be false. You know from your game board that Duway has to specialize in immigration. You also know that Duway must specialize in either employment or real estate, but it doesn't matter which one. That puts Burton or Howard in tax, and Burton and Howard in either employment or real estate. Working out the possible combinations on your game board looks like this:

9781118678237-un20016.tif

Looking at the choices, Choice (A) is wrong because B can specialize in tax if H doesn't. Choice (B) is wrong because it could be true that C specializes in employment law with D. Choice (C) must be right because D can't specialize in tax. To double-check your answer, check the last two choices. Choice (D) is wrong because H can specialize in employment, and Choice (E) is wrong because H, like B, can specialize in tax. So Choice (C) is correct.

20. D. Duway specializes in more areas of law than Cheatham.

You already know one attorney who must specialize in more areas than another: Duway must specialize in more areas than Cheatham because Duway must specialize in immigration. It looks like the answer must be Choice (D). Check the other possibilities if you have time. Burton can specialize in more areas of law than Cheatham but doesn't have to, so Choice (A) is wrong. Howard can specialize in more areas of law than Duway but doesn't have to, so Choice (B) is wrong. Duway can specialize in more areas than Howard but doesn't have to, so Choice (C) is wrong. Burton can specialize in more areas than Howard, but then Howard can also specialize in more areas than Burton, so Choice (E) is wrong. Choice (D) it is.

21. B. Burton

The way to approach this exception question is to eliminate answers that present a complete and accurate list of attorneys who can specialize in both employment and immigration. You know Duway specializes in immigration and could also specialize in employment law with Cheatham, so the correct answer isn't Choice (A). Consider Burton alone in Choice (B). You already know Burton practices immigration law. When you add Burton to employment law, you have to pair him with someone. If you pair him with either Howard or Duway, the complete list of immigration and employment specialists would also have to include either Howard or Duway because both of them also practice immigration law. You can't pair Burton with Cheatham as an employment law specialist because that violates the “if C, then D” rule. You can't eliminate Choice (B).

You can eliminate Choice (C) because Burton and Howard have to practice immigration law and easily could be the only attorneys to practice employment law. Likewise, Burton and Duway must practice immigration law and could be the two attorneys who practice employment law. Cheatham and Duway would then be the real estate attorneys, and Howard or Burton would practice tax law. Cross out Choice (D). The same is true for the combination of Howard and Duway, so Choice (E) is out. The possible combinations for each of the answer choices look like this on your game board:

9781118678237-un20017.tif

Choice (B) is the only answer you can't eliminate.

22. D. Howard specializes in tax law.

The only two areas of law that Cheatham can specialize in are employment and real estate. If Cheatham occupies those areas, so does Duway. Those two specialties with immigration give Duway the maximum three specialties, so Burton or Howard must specialize in tax. Your diagram looks like this:

9781118678237-un20018.tif

Now read the choices and see which answer could be true. Eliminate answers that must be false. Choice (A) is false because Duway can't specialize in tax. Choice (B) is false because Duway must specialize in three areas. Choice (C) is false because Howard can't specialize in employment. Choice (D) may be true because Howard can specialize in tax. Choice (E) is false because Howard can't specialize in three areas of law. Choice (D) is the only one that can be true, and it's your answer.

23. C. Burton specializes in tax law.

Consider the temporary rule this question adds. If Burton is to specialize in three fields, one of them must be immigration. You know Cheatham and Duway specialize in either employment or real estate law, so Burton can specialize in one of those but not both, which excludes Choices (A) and (B). Burton must specialize in tax law to get a third specialty. It looks like Choice (C) is the answer. Choice (E) is wrong because Howard can't specialize in tax law when Burton does. Choice (D) is wrong, too, because you don't know anything about Howard aside from the fact that he must specialize in immigration. Howard could specialize in employment law along with Burton but doesn't have to; Burton could share the employment law specialty with Duway just as easily. Choice (C) is correct.

24. C. The attorney also specializes in real estate but not in immigration.

From your original game board, you know that the only attorneys who can specialize in tax are Burton, Duway, and Howard; these three also happen to be the attorneys who must specialize in immigration. Any answer choice that doesn't have the attorney specifically specializing in immigration must be false, and because you're looking for something that can't be true, that would be the right answer. Do any choices say that the attorney doesn't specialize in immigration? Yes, Choice (C). That's the answer, but if you want to test the other propositions, you can use Burton as a hypothetical. Can Burton specialize in immigration but not in real estate? Yes, if Cheatham and Duway take real estate, so Choice (A) is wrong. Can Burton specialize in both immigration and real estate? Yes, if Cheatham and Duway take employment, so Choice (B) is wrong. Can Burton specialize in employment but not real estate? Yes, if Cheatham and Duway take real estate, so Choice (D) is wrong. Can Burton specialize in neither employment nor real estate? Yes, Cheatham and Duway can possibly take both of these specialties. Choice (C) is the only answer that can't be true.

25. C. Duway specializes in three areas: employment, immigration, and real estate.

Look for rule violations. You know Burton can't share specialties with Cheatham, and you know that Cheatham must specialize in either employment or real estate. Choices (A) and (B) have Burton specializing in both of those specialties, which is impossible. Can Duway specialize in employment, immigration, and real estate? Yes, consider Choice (C) a good prospect. Choices (D) and (E) are easy to eliminate because both Duway and Howard must specialize in immigration law, and neither answer includes immigration law in the list of their three specialties. Choice (C) is correct.

Section IV: Logical Reasoning

1. C. These soccer shoes must be the best because all the best soccer players use them, and the best players wear only the best shoes.

The question asks you to identify the argument's pattern of reasoning. You know that the reasoning is flawed because the question tells you so. So what's the flaw? This argument's reasoning is circular: The car seat is safest because the most conscientious parents bought it; they're the most conscientious because they bought the car seat that claims to be safest. No external evidence is available; each half of the argument depends on the other half to support it. So look for an answer that has two conclusions, each of which depends on the other conclusion for support.

Choice (A) is wrong. A taste test is a perfectly good indicator of a soft drink's good taste, so this reasoning isn't flawed. Choice (B) is wrong, though it may look a bit like the argument. The difference is that attending a good preschool could help kids do better in elementary school, so the conclusion in the second statement isn't circular; it's just a repetition of the reasonable conclusion in the first sentence (the two don't depend on each other). Choice (C) does duplicate the reasoning in the argument. The writer bases the claim that the soccer shoes are the best on the fact that the best soccer players use them and justifies the claim that those players are the best with the evidence that the best players wear only the best shoes. That's completely circular, so Choice (C) looks like a good answer. Choice (D) is wrong because it makes sense; the fact that technologically savvy people use a particular browser is valid evidence that a browser is good, and the argument doesn't use the fact that these people buy this browser to prove that they're technologically savvy. Choice (E) is also wrong; if the curl-enhancing lotion really works for curly haired people, then maybe it is the best. Choice (C) is the right answer.

2. A. Homeowners with interest-only mortgages won't accumulate equity based on the increasing value of their homes.

This logical reasoning question requires you to identify the assumption or, in other words, determine which statement connects the premises to the conclusion. As a general rule, assumptions are not directly stated in the premises, and you have to accept that the stated premises are true, regardless of whether there is evidence to back them up in the text.

Locate the answer choice that best links the premises to the conclusion that interest-only mortgages should be eliminated because buyers will never accumulate equity. You can eliminate Choice (B) because it mentions a concept that's too general and that appears in only the conclusion and not the premises. Choices (C) and (D) are, in essence, restatements of the premises, so they can't be assumptions that provide a link to the conclusion. Choice (E) may at first seem plausible, but notice that this assumption is about homeowners in general, not only homeowners who take out interest-only loans. The author doesn't assume that all homeowners can't afford to pay the principal on their home loans. This answer doesn't directly link the possibility of defaulting due to lack of equity to the conclusion that interest-only mortgages should be eliminated entirely. The only assumption listed on which the conclusion depends is Choice (A). If the author wants to discontinue interest-only loans because they lead to loan default due to lack of equity, he must assume that the only way to build equity is by paying loan principal and not by benefiting from increases in the home's market value.

3. A. Healthy human beings always anticipate the sounds of their own voices and differentiate them from other voices.

You may have noticed at some point in your life that it's virtually impossible to tickle yourself. If not, go ahead and try it — you'll see. Note that the question asks you to make an inference that extends the reasoning in the statements.

You're seeking an answer that logically flows from one of the premises but isn't stated directly. Choice (B), while appealing, goes too far. The passage states that anticipating the sensation of tickling is true for “normal, healthy human beings,” but others may exist who can tickle themselves. Eliminate Choice (C) because the argument doesn't allude to future research at all. Choice (D) inappropriately extends the information regarding schizophrenics (that they can't recognize their own voices) to a healthy human. Choice (E) contradicts the assertion in the passage that humans feel a heightened sense of another's touch when they simultaneously tickle themselves with another, so it can't be correct, either. Choice (A) is supported by the premise that healthy humans can't mistake their voices and provides a reason that the passage suggests (with the information in the last sentence that schizo-phrenics don't recognize their voices because they don't anticipate the sound of their voices) but doesn't state directly.

4. C. confuses the defense's failure to prove that someone else committed the crime with proof that the accused is guilty

The question asks you to spot the flaw in the argument's reasoning. What is that flaw? The prosecutor says that the defense hasn't proven that someone else committed the crime, thus proving that the accused did commit it. That's wrong. Even if you don't know about the principle of “innocent until proven guilty,” failing to show who did commit a crime isn't the same as proving that the accused did commit it. Look for an answer that shows the prosecutor's confusion of the defense's inability to furnish the proper criminal with evidence of the accused's guilt.

Choice (A) is wrong because the argument doesn't mention any evidence of the accused's innocence. Some evidence may exist, and the prosecutor may be ignoring it, but if it's not in the argument, you can't put it in an answer. Choice (B) is wrong because the prosecutor does name a flaw in the defense's arguments: The defense has failed to prove that someone else committed the crime. Whether or not that is a valid criticism is beside the point. Choice (C) is exactly what you're looking for; it looks like the right answer. Choice (D) is a bit tricky because the defense has agreed that the crime did occur, but the prosecutor isn't trying to use this admission as proof that the defendant committed the crime. Rather it's the defense's inability to pin the crime on someone else that the prosecutor provides as proof of the accused's guilt. Choice (E) is wrong. Whether the prosecutor has considered that someone else may have committed the crime isn't really the point. The prosecutor is trying to prove that the accused is guilty and has some evidence to back that up. The possibility that someone else committed the crime is irrelevant. The flaw isn't in the prosecutor's case; it's in the prosecutor's critique of the defense's case. Choice (C) is the correct answer.

5. E. As the Basin and Range area continues to sink, Phoenix, Arizona, will become drier, while areas to the north will receive more moisture.

You're tasked with drawing a conclusion from the premises. An appropriate conclusion covers all the information in the premises and doesn't include any outside information. There isn't enough information in the paragraph to draw the conclusion in Choice (A). It says Phoenix is low but still states that its elevation is 1,100 feet, which is more than 1,000 feet above sea level. The argument isn't about the expansion of the region but rather its sinking, so Choice (B) isn't right. Choice (D) addresses the compression of the Earth's crust, while the passage says that expansion causes the region to sink, so Choice (D) can't be a conclusion. Choices (C) and (E) come up with opposite conclusions, but the one that's supported by the passage is Choice (E), so eliminate Choice (C). Choice (E) correctly implies, based on the pattern described in the last sentence of the premises, that the sinking will continue in the southern area, making the climate drier and passing even more moisture to the north. Therefore, you can conclude from the premises that the southern area will continue to get drier.

6. B. Even after hundreds of years of use, names like Patrick, Seamus, and Sean are still not truly Irish.

This argument provides reasons for the choosing of Irish boys’ names. The author explains that many “Irish” names were actually imposed on the Irish by Anglo-Norman invaders. The question asks you to make an inference based on the argument.

Choice (C) mentions a fact that's stated directly in the premises, so it can't be an inference. Choice (D) just reiterates information that the author states directly, so you can knock that one out of contention, too. Choices (A) and (E) have the opposite problem; their information isn't stated in the premises, but you also don't have enough information to infer them from the premises. Concluding that Irish parents prefer the most traditional name available is too far-fetched, as is concluding that the introduction of non-Irish names to Ireland was an injustice. The only answer that works is Choice (B). If the author of the argument speaks of traditional names as those that were Irish before the 12th century, the author must think that Seamus, Sean, and Patrick are not traditional names.

7. D. Americans have universally adapted to the QWERTY keyboard and aren't interested in learning an entirely new system.

The question asks you to weaken the conclusion that everyone would buy keyboards with efficient key arrangements if they were available. To weaken this conclusion, you need to choose an answer that gives a reason why people wouldn't want the new keyboard. Choice (A) moderately weakens the conclusion by implying that the human brain is capable of adapting to the QWERTY keyboard, but it doesn't weaken the idea that consumers will flock to a more efficient arrangement. Choice (B) actually strengthens the conclusion somewhat by arguing that the justification for the QWERTY keyboard no longer exists. Choice (C) is out because the presence of more keys on the computer keyboard doesn't necessarily mean that the keys can't be arranged efficiently and that people wouldn't want to use the resulting efficient arrangement. Choice (E) neither strengthens nor weakens the argument, so you can eliminate that one, too. It may take a lot of testing to develop the new keyboard, but that doesn't prove that people wouldn't want it.

Choice (D) provides the strongest rebuttal to the notion that everyone will want the new keyboard. If people have used the QWERTY system for their entire lives, they aren't likely to want to learn a whole new keyboard arrangement and therefore won't buy the new keyboards.

8. A. The price of a gallon of gasoline that is quoted at the pump and on gas station signs includes all fuel taxes and fees.

This question requires you to weaken the conclusion that the quoted price for travel expenses isn't the final price. To weaken this conclusion, you should find an area of travel that doesn't include hidden fees. Choices (C), (D), and (E) point out scenarios in which hidden fees exist (inside and outside of travel), so they don't at all weaken the argument. Choice (B) points out the fact that some final prices are actually lower than the quoted price, but the final price is still different from the quoted price, so Choice (B) also strengthens the argument. Choice (A) points out one instance involving travel where the final price is the same as the advertised price. This finding weakens the conclusion that there isn't “one aspect of traveling where the quoted price is the final price.” Choice (A) is the best answer of the five.

9. D. The cost of cigarettes has skyrocketed in the last three years because of tax increases on big tobacco companies.

The author asserts that the drop in the number of cigarette smokers has been caused by the increase in advertisements warning against smoking. To weaken this cause-and-effect argument, find the answer that offers another reason for the declining number of smokers. Choice (A) would weaken a conclusion that increased use of antismoking aids has led to a decrease in the number of smokers, but that's not the conclusion you're supposed to weaken here. Choice (B) is out, too, because whether heavy smokers experience withdrawal is irrelevant to the decrease in total number of smokers. The argument regards cigarette smoking, not cigar smoking, so Choice (C) is irrelevant. Choice (E) confirms that cigarette smoking has decreased, but it doesn't mention anything about what has caused that decrease. The only answer that provides evidence of another cause for the decrease is Choice (D). If cigarettes are more expensive, their cost may be the cause of the decline in the number of cigarette smokers. The advertising may have had nothing to do with it.

10. A. The mandatory fitness classes will improve the physical health of Claar & Carlisle's employees.

This question requires you to identify the assumption. The assumption is often the statement that links the last premise to the conclusion, so home in on the argument's premises and conclusion.

To make this argument, the author states the premise that better physical health equals greater productivity. The author's conclusion is that Claar & Carlisle should have mandatory fitness classes, so look for the assumption that links better physical health to mandatory fitness programs. That would be Choice (A). You can eliminate Choices (B) and (D) because they require you to read too much into the argument, which doesn't necessarily rely on the assumption that these morning classes are the absolute best way to improve health and productivity. That is, you can't know that the author assumes that morning workouts are better than evening workouts. The reason the author advocates mandatory morning workouts may be that mornings are more convenient than evenings. And the argument doesn't mention nutrition classes, so you can't know how the author feels about them. Choice (E) is wrong because the argument says nothing about the value of sleep or its link to good physical health and productivity. There may be a link, but it's not made in this argument. Finally, whether employees resent mandatory programs is irrelevant to the conclusion that fitness programs would actually increase employee productivity, so you can forget about Choice (C). Stick with Choice (A).

11. C. Some former members of Rising Stars contribute to the dance school because they enjoy attending the annual showcase.

This question asks for an underlying assumption. Generally, the best answer for an assumption question is the choice that links an element of the last premise to the author's conclusion.

The last premise of this argument is that the annual dance showcase was canceled. The conclusion is that donations will go down. So find the answer choice that links the canceled concert to a decrease in donations. Eliminate Choice (A) right away because it simply restates one of the premises. An assumption by definition isn't a direct assertion. You can cross out Choices (B) and (D); both reference what will likely happen next year. Nothing in the argument points to events that will happen next year. It's concerned only with this year's possibilities. Choice (E) is backward. It links donations to the annual showcase rather than the other way around. The only answer that connects the occurrence of the holiday concert to donation amounts is Choice (C). If donations will decrease as a result of the lack of an annual showcase, there must be at least some segment of the donor population who give to the organization based on the occurrence of the showcase.

12. C. The majority of the customers who have spent in excess of $500 on a single rental since the fuel discounts began are regular customers who now make lengthier but less frequent rental arrangements.

Approach this problem methodically. First, isolate the claim you're supposed to weaken. The executives claim that the fuel discount program has been massively successful, so you're supposed to weaken the claim that the program has been successful. The basis for the claim is the increase in single rentals of $500 or more.

Eliminate answers that suggest the program has indeed been successful. Choices (A), (B), and (E) present positive outcomes of the program and therefore strengthen rather than weaken the executives’ claim.

The remaining choices are Choices (C) and (D). Choice (D) concerns only those customers who spend less than $500 per rental. It says nothing about customers who spend more. Perhaps the program has been successful because of new customers or because it has provided incentives to customers who normally spend more than $500 per rental at other companies to switch their business to Exploration. So the best answer is Choice (C). If the increase in $500 single rentals is primarily due to a switch in the rental practices of regular customers, Exploration will experience no increase in overall earnings. When the Joe Citizen changes from spending $250 on two separate rentals at Exploration to spending $500 on one single rental, the company's overall sales numbers don't change.

13. E. Parents who care for their child full time immediately after birth when the child is most vulnerable are likely to develop a stronger emotional connection with the child than parents who do not.

This question isn't particularly difficult if you keep in mind exactly what it asks for. It's often easier to answer questions that require you to find an exception by rephrasing them. In this case, you're looking for the answer that doesn't rebut the argument. It's likely the choice that reinforces the argument that paid family leave should be mandatory for new parents.

Choice (A) calls into question whether the leave would achieve the goal of creating meaningful relationships between parents and their children, while Choice (B) shows that the leave isn't necessary for establishing meaningful relationships. Choice (C) questions whether the government has the right to mandate family leave. Choice (D) casts doubt on whether the mandated leave is sufficient to realize its purpose as a decision-making tool. That leaves Choice (E), which actually strengthens the argument by reinforcing that family leave may indeed result in meaningful relationships between parents and their children.

14. A. Taxes on a scarf imported from Taiwan to the United States are less than 35 percent of the cost of manufacturing the scarf in the United States.

The point of the argument concerns saving on the cost of scarf production. Eliminate Choice (D) immediately because it concerns production time rather than production cost. Likewise, get rid of Choice (C) because it relates to job reduction, not production costs. Eliminate any choices that contradict the statement that it's more cost effective to produce scarves in Taiwan. If transportation costs wipe out the amount the manufacturer saves in production costs, manufacturing in Taiwan isn't cost effective. So Choice (B) is out. That leaves Choices (A) and (E). Both substantiate the claim that manufacturing in Taiwan is more cost effective, but Choice (E) regards labor costs, and the argument doesn't identify what component of production in Taiwan lowers the overall costs. Therefore, the best answer is Choice (A). If producing a scarf in Taiwan is more cost effective than producing it in the United States, the taxes on the scarf coming from Taiwan have to be less than the 35 percent saved by producing it there.

15. E. Music is meaningful when it is creative and inventive.

Assumption questions ask you to find the missing premise. If distinguishing among the answer choices for an assumption question overwhelms you, concentrate on the argument's premises and conclusion. The answer that links the last premise to the conclusion is usually the correct answer.

The conclusion is that music has lost much of its creativity and artistry. It bases this assertion on three premises:

Musicians focus more on making a profit than they did before digital downloads and MP3 music sharing.

Musicians focus more on attracting technologically savvy fans than they did before digital downloads and MP3 music sharing.

Musicians focus less on producing quality, meaningful music than they did before digital downloads and MP3 music sharing.

The answer that connects the idea that musicians are focusing less on making meaningful music to the conclusion that music is less creative and artistic is Choice (E). For the author to draw the conclusion that music is less creative based on the premise that music is less meaningful, the author must assume that creativity and artistry are what make music meaningful. Choices (A) and (B) don't relate to the conclusion about the innate creativity and artistry of digital music, so they can't be assumptions. Choice (C) is merely a restatement of one of the premises. The argument doesn't go into what makes music popular; its concern is the changing focus of musicians. Therefore, the author's conclusion isn't dependent on what makes music popular and Choice (D) can't be correct. Stick with Choice (E).

16. E. The ratio of male to female employees at Goodman-Horning is about the same as the ratio of male to female job applicants.

To counter the claim that it discriminates against women, Goodman-Horning would have to show that there is a reason other than gender-biased hiring practices for the discrepancy between the number of men and the number of women who work there. Choices (A) and (C) address only the current employees and therefore shed no light on the company's practices concerning prospective employees. They don't answer the question about why there aren't more women. Choice (D) could account for the discrepancy among executives, but it doesn't say anything about the difference in the number of male and female staff members. Choice (B) essentially states that similar companies have the same male to female employee ratio, but that doesn't prove that Goodman-Horning doesn't discriminate. Maybe all companies similar to Goodman-Horning discriminate against females. Choice (E) is the choice that offers another explanation for the discrepancy at Goodman-Horning. If the company has significantly fewer female applicants for its jobs, it may not have a choice to hire more women.

17. B. Motorcycle insurance rates for all motorcycle owners have risen by 5 percent over the last decade to offset the medical costs associated with head injuries incurred by motorcycle riders who fail to wear helmets.

Focus on the exact nature of the conclusion you're supposed to weaken. Notice that you're supposed to weaken the argument against helmet mandates, and the opponents to helmet requirements say that they should be able to go without wearing a helmet because their behavior doesn't potentially hurt anyone but themselves. The answer that deals a blow to that assertion is Choice (B). It reveals that others are, in fact, negatively affected because they have to pay more in insurance rates. Choices (A) and (E) provide reasons why wearing a helmet is a good idea, but they don't specifically address the reasoning of the opponents to helmet requirements, which is focused on the right to take the safety risks, not on denying risks exist. Whether governments do or don't require helmets for motorcycle riders has nothing to do with whether they should have those requirements, so Choices (C) and (D) aren't relevant in weakening the opponents’ argument. Choice (B) is best.

18. A. The level of discretion and common sense people exhibit when they use social media directly mirrors their use of the same in day-to-day personal and business interactions.

You approach questions that ask you to strengthen the argument in the same way you approach questions that ask you to weaken the argument. Isolate the argument you're supposed to strengthen and examine the method it uses to reach a conclusion.

The question asks you to strengthen the executives’ position that they have the right to request access to potential employees’ social media pages. You don't know why executives feel this way, but you do know that their opponents claim that social media pages don't reveal information that's pertinent to job performance. If you can weaken this claim, you'll strengthen the executives’ position. Look for answers that show that a correlation does indeed exist between an employee's social media pages and the way that employee performs on the job.

Choice (B) regards social media use and how that reveals something about the user, but it doesn't address access to the content of an individual's private pages, nor does it directly refer to attributes that would affect employment. Choice (C) weakens the executives’ position, so it's clearly wrong. Choices (D) and (E) don't specifically address the propriety of allowing employer access to social media pages, so they're not as good as other answers that do regard that issue. The correct answer is Choice (A) because it directly challenges the proponents’ assertion that what people post and how they behave through social media is no indication of how they'd perform on the job.

19. B. Studies have shown that increasingly greater numbers of women are choosing elective Cesarean section deliveries for cosmetic and other personal reasons.

This question draws the conclusion that the cause of the increased percentage of expensive Cesarean section procedures is that hospitals want to make money and don't care about patients. To weaken a cause-and-effect relationship, find the answer that shows that a cause for the event other than hospital greed could exist.

Look for an answer that gives another cause for the increase in Cesarean section procedures over the last few decades. You can clearly eliminate Choice (A). The argument mentions nothing about the relationship between Cesarean section and midwives. Choice (C) regards a doctor's duty to focus on a patient's best interest but says nothing about a hospital's duty to the patient, so it's out. You're not questioning the validity of the statistical increase, so Choice (D) doesn't affect the conclusion you're supposed to weaken. That leaves Choices (B) and (E). Of the two, Choice (B) presents the more solid case that another cause for the increase in Cesarean sections may exist. It's not the hospitals that are requesting the surgeries in greater numbers, but the patients. Therefore, the increase may not be due to hospital greed and inconsideration, but to the changing preferences of the women who are having the procedures. Knowing that more surgeons know how to perform Cesarean sections, Choice (E), doesn't mean that the increase isn't due to natural population increases or, in fact, to greed. Maybe more surgeons know how to perform the procedure because they want to make more money.

20. A. At the same time that businesses began imposing stiffer sanctions for employee theft, the average number of employees per business increased.

The conclusion of this weaken-the-argument question is that penalties don't work, and the proof is that the number of thefts hasn't gone down. You weaken the argument by showing that the reason for the static number of employee thefts is something other than the failure of the stiffer penalties.

Choice (B) actually strengthens the conclusion. If businesses have fewer employees overall, you'd expect the number of employee thefts to also decrease.

You can eliminate Choice (C) because it isn't specific enough to allow you to determine that a different definition of theft would affect the number of employee theft incidents (you don't know whether the more recent definition is stricter or more lenient). Choice (E) is out because it's irrelevant. Choice (D) addresses theft in general and, therefore, doesn't relate specifically to employee theft.

That leaves you with Choice (A). If employee numbers have increased, you'd expect to see a simultaneous increase in the number of employee thefts. The fact that the number of thefts hasn't increased as employee numbers increase suggests that the penalties may indeed be working because the number of thefts per employee is actually decreasing.

21. A. The women in the study who smoked marijuana were from the same village, one whose residents were of a substantially higher socioeconomic status than the rest of the country, with better access to prenatal care.

The researchers conclude that smoking marijuana during pregnancy has little effect on the health of the unborn child. They reached this conclusion because few health differences occurred between the babies of the mothers who smoked and those who didn't. Choice (A) is the answer that most seriously weakens the conclusion because it shows that the two groups had something else distinguishing them other than marijuana smoking. If the marijuana smokers were receiving better prenatal care while the others weren't, it's difficult to say whether the babies were unaffected by marijuana or whether the effects were minimized by better medical care.

Choice (B) is irrelevant. The conclusion regards the health of babies, not mothers. Choice (C) would more likely strengthen the argument because these mothers weren't just smokers but heavy smokers, and their babies still ended up fine. Choice (D) is irrelevant because the study concerned pregnant women who smoked marijuana, not tobacco. The argument doesn't discuss how damaging tobacco is. Knowing that smoking marijuana is less damaging to a fetus than tobacco doesn't tell you much about how much marijuana affects a fetus. Saying that a toothache is less painful than a kidney stone doesn't mean that a toothache isn't painful. Likewise, Choice (E) doesn't weaken the argument because the conclusion regards the health of the babies after they were born, so the differences in the stage of pregnancy at the time of the study are irrelevant given the information provided by the argument.

22. C. Students in poorer neighborhoods cannot afford the transportation costs required to attend schools that are not located nearby.

This question doesn't give you much to work with. The conclusion is that the government should implement a school voucher system because students would benefit greatly. Then it defines a school voucher system as a tuition waiver. To weaken the argument, find an answer that calls into question the premise that students would benefit. Choices (A), (B), and (D) actually strengthen the argument by showing ways that school vouchers may provide benefits. Choice (E) is irrelevant. If vouchers allow students to choose to attend any school, and public schools are better, the students can simply attend a public school where these vouchers may also apply. The only possible answer is Choice (C). If students can't afford to travel to the school they prefer, they won't be able to benefit from attending that school because the voucher doesn't cover transportation costs.

23. C. At the time of Columbus, Native Americans were much less genetically diverse than Europeans, so there were fewer possibilities of natural immunity.

You're asked to weaken Maeve's conclusion that the 95 percent death rate statistic is far too high and that a rate of 50 percent to 75 percent would be the maximum. Because Maeve relies on an analogy between the Black Death in Europe in the 14th century and the deaths among Native Americans, the best way to weaken her conclusion is to show that these phenomena really aren't analogous.

A quick scan of the answer choices should lead you to eliminate Choice (D) immediately. The argument doesn't mention sugars in milk and presents no information to connect them to smallpox or the other diseases. You can also eliminate Choice (B) because even if death rates were high in specific locations during the Black Death, Maeve's argument concerns averages, not specifics. Both Choices (A) and (E) actually strengthen Maeve's argument. If Native Americans knew more about medicine than Europeans did, they presumably should have died in fewer numbers. If most diseases can't kill more than 75 percent of their hosts without risking extinction, a rate of 95 percent would be too high. Choice (C) is the only choice that weakens the argument. Stating that Native Americans were genetically less diverse than Europeans shows that Maeve isn't comparing analogous societies, which weakens her argument.

24. D. The NCAA tournament is appropriately named because of the “madness” it creates among employees in March of every year.

This question is — despite the slightly different wording — essentially asking you to determine the most appropriate conclusion you can draw from the prompt. Remember that a conclusion encompasses all the premises and brings in no outside information. You can eliminate Choice (A) because it talks about the NCAA basketball tournament being the world's most popular sporting event, which isn't in the premises. Choice (B) also relies on outside information — in this case, details about the way people pick winners. You may agree with Choice (E), that employees shouldn't check sports scores, or with Choice (C), that employers should just indulge their employees, but either way, these statements are opinions and not appropriate conclusions to a set of primarily descriptive premises that are neither judgmental nor supportive of employees. Choice (D) is the best answer. It covers all the premises without adding outside information and therefore is the most logical answer choice to follow the original statement.

25. E. There has been an exponential increase in the number of people around the world with cellphones that could be disrupted by solar activity.

For this question, you're asked to provide a reason that would explain the paradox that weaker solar activity causes disruption in communications and power transmission for a greater number of people than did stronger such activity a decade ago. Look for an answer choice that indicates that population has increased over the decade and explains why so many more of them will be affected.

You can begin by eliminating Choice (D). The argument mentions northern lights but not in the context of disruption. You can then eliminate Choices (A) and (C) because they both tend to explain a lessening of the impact of solar activity and fail to mention the increased numbers of disrupted people. Choice (B) seems like a possible choice because it mentions the increased use of satellite signals for radio reception. However, the answer also mentions that regular radio signals are stronger than they used to be, so the effect on communications may be balanced — neither stronger nor weaker than a decade ago. The answer doesn't give you enough information to explain the “many more people” who will be inconvenienced next time. Choice (E) is the best answer because a substantial increase in cellphone use would mean that solar flares cause more disruption to more people than they did when cellphones were less popular.

Example Essay

Johnson, Stevens, & Kunam — a mid-size advertising firm seeking an experienced advertising executive to help one of its clients resonate with a younger, more tech-savvy demographic — is faced with a decision of whether to hire a bright, technologically talented up-and-comer or an older, experienced industry professional with a more traditional but less technological approach. If I were a hiring manager with Johnson, Stevens, & Kunam, I would choose Tyler, the young recent grad with experience marketing toward the firm’s key demographic, over Miranda, the older, tried-and-true account executive who may be somewhat out of touch with how to connect with younger generations.

While both options bring with them their own pros and cons, the overriding consideration is that the face of advertising has changed dramatically in recent years. Old advertising standbys like television and print advertisements simply do not reach the same numbers that they once did, particularly among the younger audiences that the firm specifically mentions that the snack food client wishes to target.

What Tyler may lack in experience he makes up for in understanding of the snack company’s target market. And while he may lack the years of professional experience that Miranda has, Miranda’s experience is in methods that no longer reach the current audience. Tyler’s experience, albeit limited, is more closely matched to the snack company’s needs. His training with an innovative firm that targets college-age adults makes him a prime candidate for connecting with the technologically savvy college crowd with whom the snack food client seeks to strike a chord. Tyler’s lack of time spent in the industry means that the firm could likely start him at a lower pay grade than his older, more experienced competition and that the firm could therefore invest its resources in providing him with the necessary training in any areas where he may lack knowledge.

Furthermore, despite Miranda’s ad experience, she is admittedly unfamiliar with digital media and new technologies, meaning that she herself would likely require additional training and at a higher pay rate in these areas just to get up to speed. All the print and TV ad awards in the world will not be effective if the target consumers are not reading newspapers and watching television.

Therefore, given that the advertising firm desires an employee who can reach out to a newer, younger, and more technologically savvy demographic, its best choice would be to take on the person who is more familiar with the client’s target demographic. Despite Miranda’s undeniably impressive resume, teaching her new technologies and advertising techniques would be an unnecessary “reinvention of the wheel” that would likely cost the firm more money in the long run than signing on fresh new talent capable of reaching the target audience from the get-go.

Answer Key for Practice Exam 3

Section I: Logical Reasoning

1. D

2. D

3. E

4. C

5. C

6. A

7. A

8. B

9. E

10. A

11. D

12. B

13. C

14. D

15. D

16. D

17. A

18. D

19. C

20. D

21. E

22. B

23. A

24. B

25. D

Section II: Reading Comprehension

1. E

2. A

3. B

4. C

5. C

6. E

7. A

8. A

9. B

10. C

11. E

12. C

13. C

14. A

15. D

16. A

17. E

18. D

19. B

20. C

21. E

22. D

23. D

24. A

25. E

26. D

27. E

Section III: Analytical Reasoning

1. D

2. B

3. A

4. E

5. C

6. D

7. B

8. E

9. A

10. B

11. D

12. A

13. D

14. D

15. E

16. C

17. D

18. B

19. C

20. D

21. B

22. D

23. C

24. C

25. C

Section IV: Logical Reasoning

1. C

2. A

3. A

4. C

5. E

6. B

7. D

8. A

9. D

10. A

11. C

12. C

13. E

14. A

15. E

16. E

17. B

18. A

19. B

20. A

21. A

22. C

23. C

24. D

25. E

tip.eps Refer to Chapter 16 for tips on computing your score.